100% found this document useful (4 votes)
3K views120 pages

Integral Calculus

This document is the table of contents for a report on integral calculus submitted by two students. It outlines 10 chapters that will cover topics like integration techniques, integration formulas, integration by substitution, definite integrals, applications of integration, centroids, iterated integrals, and triple integrals. The table of contents provides a high-level overview of the key areas and concepts that will be discussed in the full report.
Copyright
© © All Rights Reserved
We take content rights seriously. If you suspect this is your content, claim it here.
Available Formats
Download as DOCX, PDF, TXT or read online on Scribd
100% found this document useful (4 votes)
3K views120 pages

Integral Calculus

This document is the table of contents for a report on integral calculus submitted by two students. It outlines 10 chapters that will cover topics like integration techniques, integration formulas, integration by substitution, definite integrals, applications of integration, centroids, iterated integrals, and triple integrals. The table of contents provides a high-level overview of the key areas and concepts that will be discussed in the full report.
Copyright
© © All Rights Reserved
We take content rights seriously. If you suspect this is your content, claim it here.
Available Formats
Download as DOCX, PDF, TXT or read online on Scribd
You are on page 1/ 120

BICOL UNIVERSITY

COLLEGE OF ENGINEERING
MECHANICAL ENGINEERING DEPARTMENT

INTEGRAL CALCULUS

Submitted by:
Bernadette Boncolmo
Ma. Smile Glyza Granatin
BSME – 3B

Submitted to:
Engr. Eduardo Borromeo, PME
Instructor
TABLE OF CONTENTS

1 INTEGRATION
1.1 Definition
1.2 Hyperbolic Functions
1.3 Constant of Integration
1.4 Two Simple Theorems
1.5 Fundamental Formulae
2 FUNDAMENTAL INTEGRATION FORMULAS
2.1 Standard Formulas
2.2 Formula (1): Powers
2.3 Formula (2): Logarithms
2.4 Formula (3)-(3’): Exponential Functions
2.5 Formula (4)-(9): Trigonometric Functions
2.6 Transformation by Trigonometric Formulas
2.7 Formula (10)-(11): Inverse Trigonometric Functions
2.8 Formula (12): Integration by Parts
3 INTEGRATION BY SUBSTITUTION
3.1 Change of Variable of Integration
3.2 A Dimensional Check
3.3 Trigonometric Substitutions
4 INTEGRATION OF RATIONAL FRACTION
4.1 Introduction
4.2 Partial Fractions
4.3 Distinct Linear Factors
4.4 An Important Logarithmic Formula
4.5 Repeated Linear Factors
4.6 Quadratic Factors
5 DEFINITE INTEGRALS. WALLIS’ FORMULA
5.1 Definite Integrals
5.2 Change of Limits with Change of Variable
5.3 Limitations on Certain Formulas
5.4 Wallis’ Formula
5.5 Derivation of Wallis’ Formula
6 PLANE AREAS. IMPROPER INTEGRALS
6.1 Plane Areas
6.2 Substitution Suggested by the Problem
6.3 Plane Areas in Polar Coordinates
6.4 Integrable Functions
6.5 Improper Integrals
6.6 Integrals with Infinite Limits
6.7 Infinite Discontinuities of the Integrand
7 APPLICATIONS OF INTEGRATION
7.1 The General Method
7.2 Solids of Revolution: Circular Disks
7.3 Solids of Revolution: Circular Rings
7.4 Solids of Revolution: Cylindrical Shells
7.5 Miscellaneous Solids
7.6 Length of a Curve
7.7 Surfaces of Revolution
8 CENTROIDS
8.1 Density
8.2 First Moment of Mass
8.3 Centroid
8.4 Centrgid of a System of Particles
8.5 Determination of Centroids by Integration
8.6 Centroid of a Plane Area: Rectangular Coordinates
8.7 Centroid of a Plane Area: Polar Coordinates
8.8 A Theorem of Pappus
8.9 Centroid of a Solid of Revolution
8.10 Centroids of Miscellaneous Solids
8.11 Centroid of an Arc; of a Surface of Revolution
9 ITERATED INTEGRALS
9.1 Iterated Integration
9.2 Plane Area by Iterated Integration
9.3 Volume Under a Surface
9.4 Volume Found by Integration
9.5 The Double Integal
9.6 The Double Integral in Polar Coordinates
9.7 Volume in Cylindrical Coordinates
9.8 Evaluation by Inversion of Order
9.9 Evaluation by Change of Coordinate System
9.10 Area of a Surface
9.11 Surfaces of Revolution
10 TRIPLE INTEGRALS
10.1 The Triple Integral in Rectangular Coordinates
10.2 Volume as a Triple Integral
10.3 The Triple Integral: General Formulation
10.4 The Triple Integral in Cylindrical Coordinates
10.5 Heterogeneous Masses
10.6 Centroids; Moments of Inertia
10.7 Translation Theorem on Moments of Inertia
REFERENCES
1 INTEGRATION
1.1 Definition
The process inverse to differentiation is defined as integration.
d
Thus F ( x )=f ( x), we say that F(x) is an integral or a primitive of f(x) and, in symbols,
dx
we write

∫ f ( x ) dx=F ( x)
The letter x in dc denotes that the integration is to be performed with respect to the
variable x.
The process of determining an integral of a function is called integration and the
function to be integrated is calld integrand.

1.2 Hyperbolic Functions


The following fundamental properties of hyperbolic functions should be committed to
memory by the students as we shall make frequent use of them during the study of integral
calculus.

e x −e− x e x −e−x
sinh x= , cosh x=
2 2
e x −e−x
tanh x= ,
e x + e−x
e x +e− x
coth x=
e x −e− x
2 2
sech x= x −x
, cosech x= x −x
e +e e −e
cosh 2 x −sinh2 x =1, sech2 x =1−tanh 2 x
cosec h2 x=coth 2 x−1 ,sinh 2 x=2 sinh x cosh x
cosh 2 x =cosh 2 x +sinh2 x=1+2sinh 2 x=2 cosh 2 x−1.

Logarithmic values of inverse hyperbolic functions.

(i) sinh −1 x=log [ x + √ ( x 2+ 1 ) ] ;

x + √ ( x 2+ a2 )
sinh
1 x
()
a
=log [ a ]
(ii) cosh−1 x ¿ log [ x + √ ( x 2−1 ) ] ;

x + √ ( x 2−a2 )
cosh
−1 x
()
a
=log [ a ]
1 (1+ x )
(iii) tanh−1 x= log
2 (1−x ) {; }
a+ x )
tanh−1 ( ax )= 12 log {((a−x )}
,( x< a)

1 ( x+ 1)
(iv) coth −1 x= log
2 ( x−1){ ; }
( x +a)
coth −1 ( xa )= 12 log {(x−a) }, ( x> a) .
1.3 Constant of Integration
As the differential coefficient of a constant is zero, we have
d d
[ F ( x )+ C ] =f ( x ) , if F ( x )=f ( x ) ;
dx dx
therefore ∫ f ( x ) dx=F ( x ) +c .
This constant c is called a constant of integration and can take any constant value. Also
∫ f ( x ) dx is called the Indefinite integral of f(x) w.r.t. ‘x’; for by giving different values to the
constant of integration the indefinite nature is preserved.
For example, we know that
d 1
sin−1 x= and; is follows, where the omit the constant of integration, that
dx √(1−x 2)
1
∫ 2
dx is equal to – cos−1 x .
√(1− x )
But it is wrong to conclude from above that sin−1 x is qual to – cos−1 x . The correct
inference is that the two integrals, given above differ in their constant integration.
The correct result, as we see from trigonometry, is that
1
sin −1 x= π−cos−1 x.
2
The arbitrary constant of integration may be imaginary also. Generally such a constant is
added to make the result real.

1.4 Two Simple Theorems


Theorem1:
The integral of a sum of difference of a finite number of functions is equal to the sum or
difference of the integrals of the functions.
Symbolically

∫ [f 1 ( x )± f 2 ( x )± … ± f n (x )]dx
¿ ∫ f 1 ( x ) dx ±∫ f 2 ( x ) dx ±… ±∫ f n ( x ) dx .
Proof:

Let ∫ f 1 ( x ) dx=F 1 (x) ,∫ f 2 ( x ) dx=F 2 ( x), …∫ f n ( x ) dx=F n ( x ) .


d d d
Clearly { F 1 ( x ) ± F 2 ( x ) }= F 1 ( x ) ± F (x)
dx dx dx 2
¿ f 1( x )± f 2( x ) ,
Hence from the definition of the integral, we have

∫ {f 1 ( x ) ± f 2 ( x ) } dx=F1 ( x )+ F 2 ( x )
∫ {f 1 ( x ) ± f 2 ( x ) ± … ± f n ( x ) }dx

∫ f 1 ( x ) dx ± ∫ f 2 ( x ) dx ± … ±∫ f n ( x ) dx .
Theorem 2:
The integral of the product of a constant and a function is equal to the product of the
constant and the integral of the function.

Thus if λ is a constant, then ∫ λf ( x ) dx=λ ∫ f ( x ) dx.


Proof:
d
Let ∫ f ( x ) dx=F ( x ) . Then dx F ( x )=f ( x ) .
d d
By differential calculus, { λF ( x ) } =λ F ( x ) =λf ( x ) .
dx dx
∴ by definition of integral

∫ λf ( x ) dx=λF ( x )=λ ∫ f ( x ) dx .

1.5 Fundamental Formulae


We have read in differential calculus that

d x n+1 ( n+1 ) x n n
( )
dx n+1
=
(n+ 1)
=x .

n x n +1 (
Thus ∫ x dx + n+ 1
, n ≠1 ) .

The above formula is very important and shall be frequently used in this paper. This
may be remembered like this:
“To find the integral of x n w.r.t. ‘x’, increase the index (power) of x by one (unity) and divide
by the increased index.”

4 5 ( 52 )+1 7
x x 2
Thus ∫ x 3 dx= 4
,∫ x dx= 2

5
= x2 ;
7
2
+1 ()
1 −5 x −5+1 −1 −4 −1
∫ x5 dx= ∫ x dx= = x = 4;
−5+1 4 4x
−1
−1 1
1 2 x 2 +1
∫ 1
dx=∫ x dx= =2 x 2 =2 √ x;
−1
x 2
( )
2
+1

0+1
x
and ∫ dx=∫ 1 dx=∫ x0 dx= 0+1 =x .

Thus ∫ a dx=ax i.e the integral of a constant is equal to the constant multiplied by the
variable.
However if n=−1, we have

1 d 1
∫ x −1 dx=∫ 2 dx=log x , [ ∵
dx
log x=
x ]
Example (a). Integrate sec x tan x−5 cose c 2 x .

Here I = ∫ (sec x tan x−5 cose c2 x) dx


¿ ∫ sec x tan x dx−5∫ cose c2 x dx=sec x+5 cot x .

(2 cos x)
Example (b). Integrate { (3 sin2 x )
+1
}
2cos x 2cos x
Here I =∫
( 2
3 sin x )
dx=∫
3 sin 2 x
dx +∫ 1 dx

2 −2
¿ ∫ cosec x cot x dx +∫ 1 dx= cosec x+ x .
3 3
x2 x3
Example (c). Integrate 1+ x+ + +…
2 ! 3!

x2 x 3
Here (
I =∫ 1+ x+ + +… dx
2! 3 ! )
1 2 1
¿ ∫ 1 dx+∫ x dx +∫ x dx +∫ x 3 dx +…
2 6
x2 x3 x4
¿ x+ + + +…
2 6 24
Solved Problems:

( 2 x 3 +3 x−7 )
1.) Integrate 2 .
3
x
Solution:
a b
2.) Integrate + + c.
x2 x
Solution:

2
( x 2+ 8 )
3.) Integrate .
x4
Solution:

1 1−x 4
4.) Integrate 3 1−x +sec x tan x .
+
x4
Solution:

5.) Integrate

5 cos3 x+2 sin 3 x 1+2sin x 1−cos 2 x


2 2
+ √ (1+sin 2 x)+ + .
2 sin x cos x cos2 x 1+ cos 2 x
Solution:
The given expression may be written as
Now integrating, we get

6.) Integrate e x +2 sin x−3 cos x .


Solution:

2 FUNDAMENTAL INTEGRATION FORMULAS


2.1 Standard Formulas
In this chapter we shall learn how to apply the following basic formulas:
n +1
u
(1) ∫ u n du= n+1 +C , n ≠1 ,

du
(2) ∫ =ln u+ C , u>0 ,
u
du
(2’) ∫ =ln (−u ) +C1 , u<0 ,
u
du
(2’’) ∫ =ln|u|+C 2 , u ≠ 0 ,
u
(3) ∫ eu du=eu +C ,
u au
(3’) ∫ a du= ln a
+C , a> 0 ,
(4) ∫ cos u du=sin u+C ,
(5) ∫ sin u du=−cos u+ C ,
(6) ∫ sec 2 u du=tanu+ C ,
(7) ∫ csc2 u du=−cot u+C ,
(8) ∫ sec u tan u du=sec u+C ,
(9) ∫ cscu cot u du=−csc u+C ,
du u
(10) ∫ =arcsin + C , a>0
2
√ a −u 2 a

du 1 u
(11) ∫ a2 +u2 = a arctan a +C ,
(12) ∫ u dV =uV −∫ V du.
The test of the correctness of an integral is that its derivative must be the given
integrand. The above formulas are easily verified by differentiation.

2.2 Formula (1): Powers


The power formula is
n +1
u
(1) ∫ u n du= n+1 +C , n ≠1.

Example (a). Evaluate ∫ √ cos 2 θ sin 2θ dθ.

Since
d¿
we insert the factor −2 and apply (1) with u=cos 2 θ:
1
−1
∫ √cos 2 θ sin 2θ dθ= ( cos 2 θ ) 2 ¿ ¿¿
2 ∫
3
2
1 ( cos 2 θ )
¿− ∙ +C
2 3
2
3
1 2
¿− ( cos 2 θ ) + C
3

u ln 2 ( 1+u2 ) du
Example (b). Evaluate ∫ .
1+u2
An integral such as the one above should be evaluated by inspection. Note that,
essentially, the integral contains only a power of ln ⁡(1+u2) and the differential,

2 u du
d ln ( 1+u2 )=¿ ¿,
1+u2
of that quantity. Therefore the integral is basically one of the power-formula type. The result
should contain the quantity ln ( 1+u2 ) to an exponent higher by unity than the exponent in the
integrand. Hence we perform the differentiation
2u du
d ln 2 ( 1+u2 )=3 ln 2 ( 1+u2 )
1+ u2
6 u ln 2 ( 1+u 2) du
¿ .
1+ u2
The above differential agrees with the integrand except for a constant factor 6, which we know
can be adjusted. We are thus to write

u ln 2 ( 1+u2 ) du 1 3
∫ 1+u2 = 6 ln ( 1+u 2) + C .

Solved Problems:
Evaluate the following integrals; check by differentiation.
1.) ∫ cos 4 x sin x dx

Solution: Let u=sin y du=cos y dy

Thus, ∫ ( cos x ) 4 sin x dx


¿−∫ ( cos x )4 (−sin x dx )
1
¿− cos5 x+ C
5
CHECK:

1 −5
5 (
d ¿− cos 5 x +C =
5 )
cos 4 x (−sin x dx )= ( cos x )4 sin x dx

cos 2t dt
2.) ∫ = ( 1+sin 2 t )4 cos 2t dt
4 ∫
( 1+sin 2t )
Solution: Let u=1+ sin 2t du=cos 2t dt

Thus, ∫ ( 1+sin 2 t )4 cos 2t dt


1
( 1+ sin 2t )4 cos 2 t dt
2∫
¿

−3
1 ( 1+sin 2 t )
¿ +C
2 −3
1
¿− ( 1+sin 2t )−3 +C
6
CHECK:

−1 −1
d [ 6
( 1+sin 2t )−3 +C =
6 ]
(−3 )( 1+sin 2 t )4 cos t 2 dt
4
¿ ( 1+sin 2t ) cos 2 t dt
( 2 x +1 ) dx
3.) ∫
( x+ 2 )4 ( x−1 )4
Solution:
( 2 x +1 ) dx −4
∫ 4
=∫ ( x2 + x−2 ) ( 2 x +1 ) dx
[ ( x+ 2 )( x−1 ) ]
Let u=( x 2+ x−2 ) du=( 2 x+ 1 ) dx
−4
Thus, ∫ ( x 2 + x−2 ) ( 2 x +1 ) dx
−3
( x2 + x−2 )
¿ +C
−3
−1
¿ 3
+C
2
3 ( x + x−2 )
CHECK:

−1 2
[ ( x + x−2 ) C = −1 (−3) ( x 2+ x−2 ) ( 2 x+1 ) dx
]
−3 −4
d
3 3
( 2 x+ 1 ) dx ( 2 x+1 ) dx
¿ 4
=
( x2 + x−2 ) ( x +2 ) 4 ( x−1 ) 4

2.3 Formula (2): Logarithms


Next we shall apply
du
(2) ∫ =ln u+ C , u>0 ,
u
together with its associated forms
du
(2’) ∫ =ln (−u ) +C1 , u<0 ,
u
du
(2’’) ∫ =ln|u|+C 2 , u ≠ 0 ,
u
x dx
Example (a). Evaluate ∫ .
1−x 2
Formula (2) says, in words: The integral of any quotient whose numerator is the
differential of the denominator is the logarithm of the denominator. Therefore we insert the
factor −2:
x dx 1 −2 x dx 1
∫ 1−x 2 = 2 ∫ = ln (1−x 2)+C .
1−x 2
2
The integral in this example can equally well be evaluated in the following manner:
x dx x dx −1 2 x dx
∫ 1−x 2 =−∫ x 2−1 = 2 ∫ x 2−1
1
¿− ln ( x 2−1 ) +C1 ,|x|>1 ,
2
a result which can also be obtained by employing (2’) directly.
Application of (2’’) yields
x dx −1
∫ 1−x 2 = ln |1−x 2|+C 2 , x ≠ 1 , x ≠−1.
2
Each of the above three evaluations of the integral in this example has its own
restrictions, its own range of validity. In practice, specific problems usually dictate the form to
be used because the variables involved range over known intervals.
We shall ordinarily leave answers in the form to which (2) leads us. It is assumed that
the reader can convert the result to the form given by (2’) or (2’’) whenever such conversion is
necessary.

x 2−x
Example (b). Evaluate ∫ dx .
x+ 1
By division we find
2
x −x 2
=x−2+
x+1 x +1
Therefore
2
∫ xx+−x1 dx=∫ ( x−2+ x+1
2
) dx
1 2
¿ x −2 x−2 ln ( x +1 ) +C .
2
Rule. As the first step toward integrating a rational fraction, carry out the indicated
division until the numerator is of lower degree than the denominator.

Solved Problems:
Evaluate the following integrals; check by differentiation.
4 dt
1.) ∫
5 t+2
Solution: Let u=5t +2 du=5 dt
4 dt 4 5 dt
Thus, ∫ = ∫
5 t+2 5 5 t+2
4
¿ ln ¿ 5 t+2∨¿+C ¿
5
CHECK:
d¿
2
2.) ∫
( u2 +1 ) du
u3
Solution:
2
( u2 +1 ) du (u 4 +2u 2+1) 2
∫ =∫ du=∫ ( u+ +u−3)du
u 3
u 3
u
du
¿ ∫ u du+∫ +∫ u−3 du
u
u2 u−2
¿ +2 ln u+ +C
2 −2
1 1
¿ u2 +2 ln ¿u∨¿− u−2 +C ¿
2 2
CHECK:

1 2 1 2 2 du 1
d [ 2
u +2 ln |u|− u−2 +C = u du+
2 2 u 2 ]
− (−2 )−3 du

2
2 −3 (u4 +2 u2 +1) ( u 2+1 ) du
(
¿ u+ +u du=
u )
u3
du=
u3
( x+ 6 ) dx
3.) ∫
( x +2 )2
Solution:

( x+ 6 ) dx ( x +2 ) + 4 dx 4 dx
∫ ( x +2 ) 2
=∫
[( x +2 ) 2
dx=∫
x +2]+∫
( x +2 )2
dx
¿∫ +4 ∫ ( x +2 )−2 dx
x +2
( x +2 )−1
¿ ln |x +2|+ 4 +C
(−1 )
4
¿ ln |x +2|− +C
x +2
CHECK:

4 dx (−dx )
[
d ln |x +2|−
x +2
+C =
x +2 ]
−4
( x +2 )2
1 4 ( x+2 ) + 4
¿
[ +
x+ 2 ( x+2 )2
dx=
]
( x +2 )2
dx

2.4 Formula (3)-(3’): Exponential Functions


These are two basic formulas for the integration of exponential functions:

(3) ∫ eu du=eu +C ,
u au
(3’) ∫ a du= ln a +C , a> 0 ,
Equation (3’) includes (3) as a special case but (3) is used much more frequently than
the general (3’).
Example (a). Evaluate ∫ sin 2 x ecos 2 x dx .
If we insert the factor −2, this can be evaluated by (3), with
u=cos 2 x , du=−2 sin2 x dx :
−1 −1 cos 2 x
∫ sin 2 x ecos 2 x dx= 2 ∫ e cos 2 x (−2 sin 2 x ) dx=
2
e +C .

dx
Example (b). Evaluate ∫ .
32 x
Since
d u u
a =a ¿,
dx
we proceed as follows:

dx −2 x −1 3−2 x
∫ 32 x =∫ 3 dx= 2 ln 3 +C .

Solved Problems:
Evaluate the following integrals.
2
3 v −1
1.) ∫ v e dV

Solution:
2
1 2

∫ v e3 v −1 dV = 6 ∫ e3 v −1 6 V dV
1 2

¿ e 3 v −1+ C
6
1
x
2.) ∫ e dx
2
x
Solution:
1
x 1
e dx −dx
∫ x 2 =−e x x 2 ( )
1
¿−e x +C
3.) ∫ ln e2 x dx
Solution:

∫ ln e2 x dx=∫ 2 x ln e dx
¿ ∫ 2 x dx

¿ x 2+C
2.5 Formula (4)-(9): Trigonometric Functions
The basic formulas for integrations involving trigonometric functions are

(4) ∫ cos u du=sin u+C ,


(5) ∫ sin u du=−cos u+ C ,
(6) ∫ sec 2 u du=tanu+ C ,
(7) ∫ csc2 u du=−cot u+C ,
(8) ∫ sec u tan u du=sec u+C ,
(9) ∫ cscu cot u du=−csc u+C .
Notice the way in which the functions pair off for purposes of integration. The pairs sin u
and cos u, sec u, and tan u, csc u and cot u fit well together. An integral involving, for instance,
sin x and tan x is not in appropriate form for application of simple integration formulas. Upon
meeting such an integral we first put the integrand entirely in terms of sin x and cos x or in
terms of tan x and sec x.

Example (a). Evaluate ∫ sin x tan x dx.


We proceed as follows:

sin 2 x dx 1−cos 2 x
∫ sin x tan x dx=∫ cos x =∫ cos x dx
¿ ∫ sec x dx−∫ cos x dx

¿ ln ( sec x + tan x )−sin x +C


Because of the situation described above, the exercises are usually slated in terms of
appropriate pairs of functions. Such pairing permits the student to do more exercises (and thus
acquire more skill) in a specified time than would otherwise be possible. There is of course, no
guarantee of corresponding simplicity in the integrals encountered in the normal course of
events in engineering or other scientific work.
2 1
Example (b). ∫ x sin x dx= ∫ sin x 2 2 x dx
2
2
1
¿− cos x +C .
2
Example (c). ∫ tan2 θ dθ=∫ (sec2 θ−1) dθ
¿ tanθ−¿ θ+C . ¿

Solved Problems:
Evaluate the following integrals.

sin3 y dy
1.) ∫
1+ cos y
Solution:
3 2
∫ sin y dy
1+ cos y
=∫
sin y sin y dy
1+cos y
1−cos 2 y
¿∫ sin y dy
1+cos y
¿∫ ¿ ¿

¿ ∫ sin y dy+∫ cos y (−sin y dy )

cos2 y
¿−cos y + +C
2
1
2.) ∫ tan( y − π )dy
4
Solution:
1
sin( y− π ) dy
1 4
∫ tan ( y − 4 π )dy=∫ 1
cos ( y− π )
4
1
−sin( y− π )dy
4
¿−∫
1
cos ( y− π )
4

1
| (
¿−ln cos y− π +C
4 )|
2
3.) ∫ cot dy

Solution:

∫ cot 2 dy =∫ ¿ ¿
¿−cot y− y +C

2.6 Transformation by Trigonometric Formulas


Many trigonometric integrals can be evaluated after transformations of the integrand,
requiring only the most familiar trigonometric formulas. If, instead of memorizing the types
listed below, the student will observe the character of the transformations employed, he can
easily pick the requisite method in any given case.

Type I. ∫ sin m x cos n x dx , where either m or n is a positive integer.


For definiteness, let n be a positive odd integer. Writing the integral in the form
∫ sin x cos n−1 x ∙ cos x dx, and putting
m

cos 2 x=1−sin 2 x ,
we obtain a series of powers on sin x each multiplied by cos x dx. We proceed in a similar
manner when m is odd and positive.
Example (a).
∫ sin 2 x cos2 x dx=∫ sin 2 x cos2 x ∙ cos x dx
¿ ∫ sin2 x(1−sin2 x) cos x dx

¿ ∫ sin 2 x cos x dx−∫ sin 4 cos x dx


1 1
¿ sin 3 x− sin5 x+C
3 5
n n
Type II. ∫ tan x dx ,∨∫ cot x dx , where n is aninteger .

By use of the formulas

tan 2 x=sec 2 x−1 ,


these integrals reduce to forms that can be evaluated.
Example (b).

∫ tan4 x dx=¿∫ tan2 x ¿ ¿ ¿


¿ ∫ tan 2 x sec 2 x dx−∫ tan 2 x dx
1
¿ tan 3 x−∫ (sec2 x−1)dx
3
1
¿ tan 3 x−tan x+ x +C .
3
m n m n
Type III. ∫ tan x sec x dx ,∨∫ cot x csc x dx , where n is a positive even integer .

Example (c).

∫ tan2 x sec 4 x dx=¿ ∫ tan2 x sec2 x(1+ tan2 x)dx ¿


¿ ∫ tan 2 x sec 2 x dx +∫ tan 2 x sec 4 x dx
1 1
¿ tan 3 x + tan 5 x +C .
3 5
Type IV. ∫ sin m x cos n x dx , where both m∧n are positive even integers .
When m and n are both even, it is easily seen that the method used for Type I is useless.
Instead, we use the formulas
1 1
sin 2 x= ( 1−cos 2 x ) ,cos 2 x= ( 1+cos 2 x ) ,
2 2
1
sin x cos x= sin 2 x ,
2
repeatedly if necessary.
Example (d).

∫ sin 4 θ cos2 θ dθ=∫ ¿ ¿ ¿


1
¿ ∫ sin2 2θ ¿
8
1 1
sin 2 2θ dθ− ∫ sin 2 2 θ cos 2 θ dθ
8∫
¿
8
1
16 ∫
¿ ¿¿

1 1 1
¿ θ− sin 4 θ− sin 3 2θ+ C .
16 64 48

Solved Problems:
Evaluate each of the following integrals.

1.) ∫ cos7 θ dθ
Solution:

∫ cos 7 θ dθ=∫ cos6 θ cos θ dθ


¿∫ ¿ ¿

¿ ∫ ( 1−3 sin 2 θ+3 sin 4 θ−sin6 θ ) cos θ dθ


3 1
¿ sin θ−sin3 θ+ sin 5 θ− sin 7 θ+C
5 7
2.) ∫ tan3 θ dθ
Solution:

∫ tan3 θ dθ=∫ tan2 θ tan θ


¿ ∫ ( sec 2 θ−1 ) tan θ dθ
sinθ dθ
¿ ∫ tanθ sec 2 θ dθ−∫
cos θ
tan 2 θ
¿ + ln |cos θ|+C
2

sin 5 t dt
3.) ∫
cos2 t
Solution:
2
sin 5 t dt ( 1−cos 2 t ) sin t dt
∫ cos2 t =∫ cos2 t
( 1−2 cos2 t+ cos4 t ) sint dt
¿∫
cos2 t

¿ ∫ [ ( cos t )2−2+cos 2 t ] sint dt


¿−∫ [ ( cos t )2−2+cos 2 t ] (−sint dt )

( cos t )−2
¿− [ (−1)
1
−2cos t + cos 3 t +C
3 ]
1
¿ sec t +2 cos t− cos3 t+ C
3

2.7 Formula (10)-(11): Inverse Trigonometric Functions


Consider next the two formulas:
du u
(10) ∫ =arcsin + C , a>0
2
√ a −u 2 a

du 1 u
(11) ∫ a2 +u2 = a arctan a +C .
In applying (10), it is important to note that the numerator du is the differential of the
variable quantity u which appears squared inside the square root symbol.
dx
Example (a). Evaluate ∫ .
√ 9−4 x 2
The presence of a constant minus the square of a variable under the square root sign is
what suggests the use of formula (10). Therefore we mentally put the quantity under the radical
into the form of the square of a constant minus the square of a variable. That is, we think of
√ 9−4 x 2 as √ 32−¿ ¿. This shows that the u in the formula (10) is to be 2x. Hence du = 2dx and
we need to insert the constant 2 into the numerator before we can employ (10). To insert the
constant legitimately, we must compensate for it by putting its reciprocal as a factor outside the
integral. Therefore we write
dx 1 dx
∫ 2
= ∫
2 √ 9−¿ ¿ ¿
¿
√ 9−4 x
1 2x
¿ arcsin +C .
2 3
Example (b).
dy dy
∫ 9 y 2 +6 y +5 =∫ ( 3 y +1 )2 + 4
1 3 dy
¿ ∫
3 (3 y +1 )2 +4

1 1 3 y +1
¿ ∙ arctan +C
3 2 2
1 3 y +1
¿ arctan +C .
6 2
Example (c).
dx
dx x
∫ 2
=∫
x √ 1−4 ln x √ 1−4 ln2 x
dx
2
1 x
¿ ∫
2 √ 1−4 ln 2 x

1
¿ arcsin ¿
6

Solved Problems:
Perform the indicated integrations.

e2 x
1.) ∫
√ 9−4 e 4 x
Solution:

e2 x 1 2 e2 x 2 dx
∫ = ∫
√ 9−4 e 4 x 4 ( 3 )2−( 2 e 2 x )2

1 2 e2 x
¿ arcsin +C
4 3
( y +4 ) dy
2.) ∫
y 2+ 16
Solution:
( y +4 ) dy 1 2 y dy dy
∫ = ∫ 2
y + 16 2 y +16
2
+4 ∫
16+ y 2
1 y
¿ ln | y 2 +16|+ arctan +C
2 4
sec 2 x dx
3.) ∫
√ 5−sec 2 x
Solution:

sec 2 x dx sec 2 x dx
∫ =∫
√ 5−sec 2 x √ 4+1−sec2 x
sec 2 x dx
¿∫
√ 4−tan2 x
sec 2 x dx
¿∫
√( 2 )2− ( tan x )2
¿ arcsin ( tan2 x )+C
2.8 Formula (12): Integration by Parts
From the formula for the differential of a product,
d ( uV )=u dV +V du ,
we find, integrating both sides,

uV =∫ u dV +∫ V du .
Transposing, we obtain the formula

(12) ∫ u dV =uV −∫ V du.


Integration by this formula is called integration by parts.
Example (a). Evaluate ∫ x sin 2 x dx
Let u=x , dV =sin 2 x dx ,
−1
du=dx ,V = ∫ sin 2 x dx= cos 2 x .
2
(It is a fact, which should be verified by the student, that in evaluating ∫ dv=v , the constant of
integration may be omitted, since the final result is the same with or without it.) Hence
−1 1
∫ x sin 2 x dx= x cos 2 x + ∫ cos 2 x dx .
2 2
1 1
¿− x cos 2 x+ sin2 x +C .
2 4
Only by experience and practice can one develop skill in telling when integration by
parts is indicated. Further, when it has been decided to try the method, no rules can be laid
down telling how to choose u and dv except that dv must be chosen so that ∫ dv can be
evaluated. However, in integrating a product, this method gives us a chance to differentiate one
of the factors. In Example (a), differentiating x, we replace it by 1; differentiating sin x, we
replace it by cos x. The former change, being more drastic, seems more promising. By looking
ahead a bit in this way, we can usually make the right choice in the first instance.
Example (b). Evaluate ∫ sec 2 θ dθ .

Take u=sec θ , dv=sec 2 θdθ ,


du=sec θ tan θ , v=tan θ

∫ sec3 θ dθ=secθ tan θ−∫ sec θ tan2 θ


¿ secθ tanθ−∫ sec 3 θdθ+∫ sec θ dθ
Evaluate the last integral and transpose the next-to-last to the other side:

2∫ sec 3 θ dθ=sec θ tan θ+ ln(¿ sec θ tan θ)+C , ¿


1 1
∫ sec 3 θ dθ= 2 sec θ tanθ+ 2 ln (¿ sec θ tanθ)+C 1 . ¿
≈ Evaluate ∫ e x 2 x dx .
Take u=e x ,dV =sin 2 x dx ,
−1
du=e x dx ,V = cos 2 x ;
2
−1 x 1
(1) ∫ e x sin 2 x dx= 2
e cos 2 x+ ∫ e x cos 2 x dx .
2
Since this new integral is no simpler than the original, let us return to the given integral and
take

u=sin 2 x , dV =e x dx ,
du=2 cos 2 x dx ,V =e x ;
(2) ∫ e x sin 2 x dx=e x sin 2 x−2 ∫ e x cos 2 x dx .
Here again we have a failed temporarily, but since the troublesome integral is exactly the same
one that appear in (1), it may be eliminated from the two equations; multiplying each member
of (1) by 4 and adding it to the corresponding member of (2), we find that

5∫ e x sin2 x dx=−2 e x + e x sin2 x+ C ,


−2 x 1
∫ e x sin 2 x dx= 5
e cos 2 x + sin 2 x +C 1 .
5
Before considering himself skilled in the use of integration by parts, the student should
learn to dissociate the technique from the letters employed in formula (12). The original
integrand is always split into two factors, one of which involves a differential. Let us place those
factors beside one another. Then beneath the factor with the differential put its integral;
beneath the factor with no differential put its differential. Thus, in attacking the integral

∫ arcsin u du
we form the array shown below.

arcsin u du The components of the lower line are the factors in the new
integrand. The integrated portion in formula (12) is the product of
du u the two components which contain no differential. Therefore we
√1−u2 write

u du
∫ arcsin u du=u arcsin u−∫
√ 1−u2
1
2 2
¿ u arcsin u+ ( 1−u ) +C .

Solved Problems:
Perform the integrations on the following integrals.

1.) ∫ ln x dx
Solution:
dx
Let u=ln x du=
x
dV =dx V =x

Thus, ∫ ln x dx=x ln x−∫ x ( dxx )


¿ x ln x +C

1
2.) ∫ x 3 ( a2 + x 2 ) 2 dx

Solution:
Let u=x2 du=2 xdx
1 1 3
1 1
dV =( a + x ) x dx V = ∫ ( a2 + x 2 ) 2 2 x dx= ( a2 + x 2 ) 2
2 2 2
2 3
1 3 3
Thus, ∫ x ( a + x ) dx = 13 ( a2 + x 2 ) 2 − 13 ∫ ( a 2+ x 2 ) 2 2 x dx
3 2 2 2

5
3 2 2 2
1 2 2 1 (a + x )
2 2
¿ x (a + x ) − +C
3 2 5
2
3 5
1 2
¿ x 2 ( a2+ x2 ) 2 − ( a2 + x 2 ) 2 +C
3 15
3.) ∫ x √ x +3 dx

Solution:
Let u=x du=dx
3
1
( x +3 ) 2
2
dV =√ x +3 dx V =∫ ( x +3 ) dx=
3
2
3 3
2 2
Thus, ∫ x √ x +3 dx= x ( x+ 3 ) 2 −∫ ( x+ 3 ) 2 dx
3 3
5
3 2
2 ( 22 ( x+ 3 )
¿ x x +3 ) − ∫ +C
3 3 5
2
3 5
2 2 4 2
¿ x ( x +3 ) − ∫ ( x +3 ) +C
3 15
3 INTEGRATION BY SUBSTITUTION
3.1 Change of Variable of Integration
Many integrals may be evaluated by introducing a new variable of integration, say x, the
two variables being connected by some suitable formula. The change of variable is usually
brought about by means of an explicit substitution

x=φ ( z ) dx=φ ' ( x ) dx .


This process, called integration by substitution, is highly important. It is to be remembered that
not merely x, but dx as well, must be replaced by the proper expression in terms of the new
variable.
The substitution to be made must be determined by inspection of the integrand. No
general rules can be given; skill in the choice of substitutions comes only with practice. There is
however, one rather crude rule of thumb, which succeeds often enough to make it worth
consideration. Determine, if possible, what quantity seems to be causing the trouble (keeping
you from performing the integration by the simple devices of Chapter 2) and then introduce a
new variable for that quantity. In many cases, several different substitutions may be found, any
one of which will succeed.

Example (a). Evaluate ∫


√ x dx
1+ x
Put √ x=z . Then x=z 2∧dx =2 zdz . Hence

√ x dx =2 z 2 dz =2
∫ 1+ x
∫ 1+ z 2 ∫ (1− 1+1z ) dz
2

¿ 2 z−2 arctan z+C


¿ 2 √ x−2arctan √ x +C .
z3 d2
Example (b). Evaluate ∫
√ z2 −a2
Put √ z 2−a2=v . Then z 2=v 2+ a2∧z dz=v dv . Since z dz is expressed simpy (as v dv) in
terms of the new variable, it is wise to exhibit z dz explicitly before proceeding with the
substitution. Therefore we write

z3 d2 z 2 dz ( v 2+ a2 ) vdv
∫ =∫ =∫ v
√ z2 −a 2
√ z 2−a2
1
¿ v 3 + a2 v+ C
3
2 1
1
¿ ( z 2−a 2) 3 + a2 ( z 2−a2 ) 2 + C .
3
At time it is desirable to put the result in other forms. From the above we obtain

z3 d2 2

∫ 2 2 = 13 ( z 2−a 2 ) 3 [ z 2−a 2+3 a 2 ]+C


√ z −a
1
1
¿ ( z 2+ 2 a2 )( z2 −a2 ) 2 +C .
3
Integrals involving √ a2−x 2, √ a2 + x 2, √ x 2−a2 occur very often. It should be noted that
substitution of a new variable for the radical, as in Example (b), is indicated whenever the
integrand contains, as a factor, an odd positive or negative integral power x; but if not, the
radical will reappear after the substitution.
It will be found that some of the integrals in this chapter can be solved directly by the
methods of Chapter 2. Although substitutions are frequently necessary, the student should be
alert for opportunities to avoid them by exercise of a little ingenuity. For instance, the integral in
Example (b) above is easily evaluated without recourse to a substitution. Write

z3 d2 ( z2 −a2 +a2 ) z dz
∫ =∫
√ z2 −a 2
√ z 2−a2
1 −1
2 2 2 2 2 2 2
¿ ∫ ( z −a ) z dz+ a ∫ ( z −a ) dz , etc .

Solved Problems:
Evaluate the following integrals.

1.) ∫ sin √ t dt

Solution:
Let y= √ t∨ y 2=t dt =2 y dy
Thus, ∫ sin √t dt=∫ sin y 2 y dy=2∫ ¿ ¿
Let u= y du=dy
dV =sin y V =−cos y

[
Hence, 2∫ y sin y dy=2 − y cos y +∫ cos y dy + C ]
¿ 2 [ sin y − y cos y ] +C
Since y= √t , Then

∫ sin √t dt=2 [ sin √ t−√ t cos √ t ]+C


x2 −a2
2.) ∫ √ dx
x
Solution:
Let u=√ x 2−a2∨u2=x 2−a2 udu=x dx
2 2
u2 du
Thus, ∫ √ x x−a dx =∫
u udu
=∫
√ u2 +a2 √ u2 +a2 u2 +a2
a2 1 u
(
¿ ∫ 1− 2
u +a)2 [a ]
du=u−a 2 arctan +C
a

u
¿ u−a arctan + C
a
Since u=√ x 2−a2, Then

√ x2 −a2 2 2 √ x 2−a2
∫ x
dx = √ x −a −a arctan +C
a
dx
3.) ∫ 2 2
x ( x −a )
Solution:
1 −du
Let x= dx = 2
u u
−du
u2
Thus, ∫ x (xdx
2
−a2 )
=∫
1 1
=−∫
udu
1−a2 u 2
(
u u2
−a
)
2

2
1 (−2 a udu) 1
¿ 2∫ 2 2
= 2 ln |1−a2 u 2|+C
2a 1−a 2 2a
1
Since u= , Then
x

dx 1 a2
∫ x (x 2−a2 ) 2 a2
= ln 1−
x
2
+C| |
3.2 A Dimensional Check
In Example (b) of the preceding section, let z and a each represent a length. Then dz is
also a length and the integral

z3 d2
(1) ∫
√ z2 −a2
has the dimension (length)3. In every step of the work in evaluating (1), each term must have
the dimension (length)3. This dimensional property should be used to check the work. Any term
not having the proper dimension is incorrect, a fact which can be proved with the aid of the
concept of homogeneity.

3.3 Trigonometric Substitutions


Many integrals can be evaluated by substituting a trigonometric function x. The
following substitutions are especially promising:
(1) When the integrand involves a 2−x 2 , lry x=a sin θ .

(2) When the integrand involves a 2+ x 2 ,lry x =a tan θ .

(1) When the integrand involves x 2−a 2 , lry x=a sec θ .


However, it will be found that these combinations by no means exhaust the usefulness of
trigonometric substitutions.
Let us examine the reasons underlying the choice (1) above. We know that

1−sin2 θ=cos 2 θ.
Therefore, if x is chosen to be a sin θ ,

a 2−x 2=a2−a2 sin2 θ=a2 cos2 θ.


We thus replace (a 2−x 2) by a single term a 2 cos 2 θ . Furthermore, that single term is a perfect
square, which is particularly effective when the quantity (a 2−x 2) appears under a square root
symbol. Corresponding analyses of the choices (2) and (3) above should be made by the student.
dx
Example (a). Evaluate ∫ 2
( a 2−x 2) 3
Putting x=a sin θ , dx=a cos θ dθ , we get
a cos θ dθ 1 cos θ dθ 1 cos θ dθ
∫ 2
=
a 2∫ 2
= 2∫
a cos 3 θ
( a 2−a2 sin2 θ ) 3 2
( 1−sin θ ) 3

1 1
¿ 2∫
sec 2 θ dθ= 2 tan θ+C .
a a

From the triangle,


x
tanθ= ,
√ a −x 2
2

whence
Figure 3.1 dx x
∫ 2
= +C
.
a √ a2−x 2
2
( a 2−x ) 2 3

The triangle in Fig. 3.1 was constructed, of course, to permit easy return to the original
variable x. Note that the quantity √ a2−x 2, which originally suggested the substitution used,
appears in the triangle. This brings in a mild check on the work. Unless the quantity, which
suggests a trigonometric substitution, appears in the associated triangle relating new and old
variables, there is probably an error in the work.

√ x−a dx
Example (b). Evaluate ∫ 3 .
2
x
Here there are two troublesome elements; both ( x−a) and x appear with fractional
exponents. If we put x=u2 , the disturbing element (u2 −a) would appear under a square root
sign. The choice x−a=v 2 would introduce ( v 2+ a) for x, which is undesirable in the same way.
2 2
Now ( x−a) may be thought of as ( √ x ) −( √ a ) , thus suggesting the substitution in (3)
above. Hence we try

√ x=√ a sec θ
from which

x=a sec 2 θ , dx=2a sec 2 θ tanθ dθ .


Then
2 2
∫ √ x−a dx=∫ √
a sec θ−a ∙ 2 a sec θ tan θ dθ
3 3
x2 a 2 sec 5 θ

2 √ a sec 2 θ−1 ∙ tan θ dθ


¿
a
∫ sec 3 θ
2
¿ ∫ tan 2 θ cos 3 θ dθ
a
Figure 3.2
2
¿ ∫ sin 2 θ cos θ dθ
a
3
2
2 3 2 ( x−a )
¿ sin θ+C= 3
+C .
3a 3a 2
x

Solved Problems:
Evaluate the following integrals.

1.) ∫ √a 2−x 2 dx
Solution: Let x=a sin θ dx=a cos θ dθ

Thus ∫ √a 2−x 2 dx=∫ √a 2−a2 sin2 θ a cos θ dθ


−∫ a 2 cos 2 θ dθ=a2∫ ( 1+cos2 θ 2 θ ) dθ
1 2 a2
¿ a θ+ sin 2 θ+C
2 4
1 2 a2
¿ a θ+ sin θ cos θ+C
2 4
x
Since sin θ= ,
a

a2−x 2
cos θ= √
a
Thus,

2 1 2 2 x 1 2 x √ a 2−x 2
∫ √a −x dx= 2 a arcsin a + 2 a a2 +C
1 x 1
¿ a2 arcsin + x √ a 2−x2 +C
2 a 2
dw
2.) ∫ 2
w √ w 2 + a2
Solution: Let w=a tan θ dw=a sec 2 θ dθ
a sec 2 θ dθ
Thus, ∫ 2 2 2 ∫ a2 tan2 θ a sec 2 θ = a12 ∫ sec
dw
=
θ dθ
tan 2 θ
w √w +a
1 cos θ dθ 1
¿ 2∫ 2
= 2 ∫ ( sin θ )−2 cos θ dθ
a sin θ a
1 (−1) −1
¿ 2
+C= 2 csc θ+C
a sin θ a
w a2 + w2
Since tanθ= csc θ= √
a w

Therefore,

dw −1 √ a2 + w2
∫ =
a2 w
+C
w 2 √ w 2+ a 2

dx
3.) ∫ 2
( Put x=2 a sin 2 θ)
x √ 2 ax−x
Solution: Let x=2 a sin2 θ dx=4 a sin θ cos θ dθ
dx 4 a sinθ cos θ dθ
Thus, ∫ 2
=∫
2 a sin θ √ 4 a 2 sin2 θ−4 a 2 sin 4 θ
2
x √ 2 ax−x
4 asin θ cos θ dθ 1
¿∫ = ∫ csc 2 θ dθ
2a sin θ 2 a sinθ cos θ a
2

1
¿− cot θ+C
a

Since sin θ= √x cot θ= √


2 a−x
√2 a √x

Therefore,

∫ dx
=
− √ 2 a−x
+C= √2 ax−x 2 + C
x √ 2 ax−x 2 a√x ax
4 INTEGRATION OF RATIONAL FRACTIONS
4.1. Introduction

The first step in dealing with an integral of this type is to carry out the indicated division
until the numerator is of lower degree than the denominator.
In this chapter, whenever the quantityax 2 +bx +c occurs, it will be assumed that
b 2−4 ac< 0. If b 2−4 ac ≥ 0, the quantity ax 2 +bx +c can be factored into real linear factors.
We can integrate fractions of the forms,
A A (2 ax +b) A
n
, n
, 2
2
( ax+ b ) ( ax +bx+ c ) ( ax + bx+ c )
The first two lead to powers, if n>1, to logarithms, if n=1; the third leads to an arctangent. We
can also integrate,
A
n
n>1
2
( ax + bx+ c )
By trigonometric substitution.

4.2. Partial Fractions

It is shown in algebra that every rational function whose numerator is of lower degree
that the denominator can be broken up into so-called partial fractions of the exact forms listed
above. It follows that every rational fraction can be integrated in elementary terms. In the next
few pages we show how to effect the breakup into partial functions.
In order to apply the results, it is necessary that the operator actually be able to find the
linear and quadratic factors of the denominator – conceivably a formidable task. Fortunately,
most cases that arise are relatively simple.

4.3. Distinct Linear Factors

The simplest case is that in which the denominator can be broken up into real linear
factors, none of which is repeated. In this case we may always rewrite the given fraction
(provided the numerator is of lower degree than the denominator) as a sum of fractions whose
numerators are constant and whose respective denominators are the factors of the original
denominator.
x 3 +2
Example(a). Evaluate ∫ dx .
x 3−x
By division,
x 3 +2 x+ 2
3
=1+ 3
x −x x −x
The factors of the denominator are x , x +1 , x−1. Assume
x+2 A B C
= + +
x −x x x +1 x−1
3

where A, B, C are constants to be determined. Clearing of fractions, we find


x +2= A ( x 2−1 ) + Bx ( x−1 ) +Cx ( x +1 )
This relation must hold for all values of x. hence, assigning to x any three values whatever, we
must obtain three simultaneous equations to determine A, B, C. but the most convenient values
to use are 0, -1, 1 (the zeros of the original denominator), for each of these causes two terms to
drop out:
x=0 A=−2
1
x=−1 B=
2
2
x=1 C=
3
Thus,
x 2 +2 2 1 1 3 1
∫ 2
x ( x −1 )
dx=∫ 1− + ∙ ( + ∙
x 2 x +1 2 x −1
dx )
1 3
¿ x 2−2 ln x+ ln ( x +1 ) + ln ( x−1 ) +C
2 2
The student is urgently warned not to forget the preliminary division (when necessary).
Without that, the above process will determine values of A , B , C ; but the sum of partial
fractions thus found will be equal to the given fraction for no values of x except the three that
were assigned.
Careful scrutiny of the method used in the Example shows that the expansion is easily
obtained mentally. Let us develop the idea in detail.
Consider any rational fraction with numerator of lower degree than the denominator,
and with denominator consisting of distinct linear factors only. Let ( x−a) be a representative
f (x)
factor of the denominator. Then the fraction may be written , where g(a) ≠ 0. The
( x−a ) g( x)
theory of rational fractions shows that,

f (x) A
= +φ ( x ) Equation 1
( x−a ) g ( x ) x−a
where φ ( x ) is the sum of the other terms in the desired expansion.
Multiply each term of (1) by ( x−a), thus getting
f ( x)
= A+ ( x−a ) φ ( x ) ,
g(x)
from which,
f ( a)
A=
g (a )
A
Thus, the numerator of the representative term can be obtained from the original
x−a
fraction by (mentally) removing the factor ( x−a ) and evaluating what remains at x=a .
x2 +1
Example(b). Expand into partial fractions.
( x−2 )( x−1 ) ( 2 x+1 )
We know that
x2 +1 A B C
= + + ,
( x−2 )( x−1 ) ( 2 x+1 ) x−2 x−1 2 x +1
from which
x 2+1 5
A= [ ( x −1 )( 2 x+1 )
2
] x av 2
=
( 1) ( 5)
=1

x +1 2
B= [ ( x−2 ) ( 2 x +1 )
x 1
] =
(−1 )( 3 ) 3
av
−2
=

5
2
x +1 4 1
C=
[ =
] ( )( )
( x−2 ) ( x −1 ) x −2 −5 −3 3
1
av
=

2 2
Therefore,
−2 1
x2 +1 1 3 3
= + + ,
( x−2 )( x−1 ) ( 2 x+1 ) x−2 x−1 2 x +1
all of which should be accomplished mentally.
4.4. An Important Logarithmic Formula

dx
To evaluate ∫ , use the method of Example(b), to get
a −x2 2

dx 1 dx 1 dx
∫ a2 −x2 = 2 a ∫ a+ x + 2 a ∫ a−x
1 1
¿ ln ( a+ x)− ln ( a−x ) +C
2a 2a
or
dx 1 a+ x 1 x−a
∫ a2 −x2 = 2 a ln a−x +C 1= 2 a ln x+ a + C'1
As a corollary, by changing signs we get,
dx 1 a−x 1 x−a
∫ a2 −x2 = 2 a ln a+ x +C 1= 2 a ln x+ a + C'1

Solved Problems:

( x−1 ) dx
1. ∫ x 2+5 x +6
Solution:

( x−1 ) A B
Let = +
x +5 x +6 x+ 2 x+3
2

∴ x−1= A ( x+ 3 ) +B ( x+ 2)

If x=−2 ,−2−1= A (−2+3 ) ∴ A=−3

If x=−3 ,−3−1=B (−3+2 ) ∴ B=4

Thus,

( x−1 ) dx dx dx
∫ x 2+5 x +6 =−3∫ x+ + 4∫
2 x +3

( x−1 ) dx
∫ x 2+5 x +6 =−3 ln|x+2|+ 4 ln|x+3|+C answer

dx
2. ∫ x 2+ ax
Solution:

1 A B
Let = +
2
x + ax x x +a

∴ 1= A ( x+ a ) + Bx
1 1
If x=0 , A= If x=−a , B=
a a

And so,

dx 1 dx 1 dx
∫ x 2+ ax = a ∫ − ∫
x a x +a

1 1
¿ ln x− ln ( x+ a ) +C
a a

dx 1 x
∫ x 2+ ax = a ln x +a +C answer

4.5. Repeated Linear Factors

If the denominator contains a factor ( x−a )r, the above method fails, since there would
be r partial fractions with the denominator x−a , and these could be combined into a single
fraction with denominator x−a . In this case, corresponding to the factor ( x−a )r, we assume
r ∂ fractions of the form
A B D
+ + …+
x−a ( x−a )2 ( x−a )r

x 3−1
Example . Evaluate ∫ dx .
x ( x +1 )3
Assume
3
x −1 A B C D
3
= + + + Equation1
x ( x+ 1 ) x x +1 ( x +1 ) ( x+ 1 )3
2

x 2−1= A ( x +1 )3 +Bx ( x +1 )2 +Cx ( x+ 1 )+ Dx Equation 2


To get the necessary four equations for the denominator of A , B , C , D , two methods are
at once available. Specific values of x can be use in the identity Equation2, or the coefficients of
like powers of x in the two members of Equation2 can be equated.
We naturally employ whatever combination of these methods yields simple equations to
be solved for the unknowns A , B , etc.
From Equation2, we obtain equations as follows:
x=0−1= A ,
x=−1−2=−D ,
Coefficients of x 3 :1=A + B
Coefficients of x 2 : 0=3 A+2 B+ C .
These equations yield A=−1 , B=2, C=−1 , D=2 , whence
x 3−1 2 1 2
∫ x ( x +1 )3 dx=∫ −1 +
( − +
x x+ 1 ( x +1 ) ( x+1 )3
2
)
dx

1 1
¿−ln x+ 2 ln ( x +1 ) + − +C .
x +1 ( x+1 )2
The algebra may be checked by obtaining equation form the identity Equation2. For
instance,
x=1 :0=8 A +4 B+2 C+ D ,
which must also be satisfied by the A , B , C , D ,if they are correct.

Solved Problems:
2
1. Integrate ∫ x −x+ 1
¿¿
Solution:

x 2−x+ 1
∫ ¿¿
A¿

r =−1: A (0)+ B (0)+C=3⟶ C=3


x=0 : A( 1)+ B(1)+C= A +B+ 3=1 → A+ B=−2 :
x=1 : A (4)+B (2)+C=4 A+2 B+ 3=1 →2 A+ B=−1

It follows that B=−3 and A=1.

x 2−x+ 1 ¿ ∫ ¿¿ ln ⁡∨ x +1∨−3 ¿ ¿ x 2−x+ 1


∫ ∫ ¿¿
¿¿

x3 + 4
2. Integrate ∫ dx
( x 2−1 ) ( x 2+3 x +2 )
Solution:

x3 + 4 x 3+ 4 3
∫ dx=∫ dx ¿ ∫ x + 4 ¿ ∫ ¿
2 2
( x −1 ) ( x +3 x +2 ) ( x +1)(x −1)(x +2)( x+1) ¿¿

A(x +1)(x−1)+ B( x−1)(x+ 2)+C ¿

−3
let x=−1 : A ( 0 ) + B (−2 )+ C ( 0 ) + D ( 0 )=3 → B=
2
5
let x=1: A ( 0 )+ B ( 0 ) +C (12 )+ D ( 0 ) =5 →C= let
12
4
x=−2 : A ( 0 ) + B ( 0 ) +C ( 0 ) + D (−3 ) =−4 → D=
3
5 4
let x=0 : A (−2 ) + B (−2 )+ C ( 2 ) + D (−1 )=−2 A +3+ − =4
6 3

−3 3 3
A= ¿ ∫ ¿¿ ∫ ¿ ¿−( )ln ⁡∨x+1∨−( )¿ ¿
4 4 2
3 +3 5 +4
¿− ln ⁡∨x+1∨ + ln ⁡∨ x−1∨ ln ⁡∨ x +2∨+ C answer
4 2( x +1) 12 3

4.6. Quadratic Factors

Corresponding to a factor in the denominator of the form a x 2+ bx+ c with


A ( 2ax +b )+ B
b 2−4 ac< 0, we assume the partial fraction , where A and B are to be determined.
a x 2 +bx +c
x 2+ 4 x +10
Example ( a ) . Evaluate ∫ dx .
x 3+ 2 x 2 +5 x
Assume
x 2+ 4 x +10 A B ( 2 x +2 ) C
3 2
= + 2 + 2 ,
x +2 x +5 x x x + 2 x +5 x +2 x +5
x 2+ 4 x +10= A ( x3 +2 x 2+5 x ) + Bx ( 2 x +2 ) +Cx .
Put x=0 :5 A=10 , A=2.
2 −1
Equate coefficients of x : A+ 2 B=1 , B= .
2
Equate coefficients of x :2 A+2 B+C=4 ,C=1.
Therefore,
2
∫ xx3++24xx2+10
+5 x
dx=∫
2 1 2 x+ 1
(
− ∙ 2 + 2
1
x 2 x +2 x +5 x + 2 x +5
dx )
1 1 x+ 1
¿ 2 ln x− ln ( x 2+2 x +5 ) + arctan +C .
2 2 2
The case of repeated quadratic factors occurs less often. Corresponding to a factor
2 r
( a x +bx +c ) , we assume r partial fractions with linear numerators as above, and successive
denominators building up step-by-step just as in,
A A (2 ax +b) A
n
, n
, 2
( ax+ b ) ( ax 2 +bx+ c ) ( ax + bx+ c )

Solved Problems:
xdx
1. ∫ x 2+ 6 x +13
Solution:

x A ( 2 x+ 6 ) B
Let 2
= 2 + 2
x +6 x +13 x +6 x +13 x +6 x +13

x= A ( 2 x+6 ) +B

1
If x=−3 , B=−3 If x=0 , 6 A−3=0 ∴ A=
2

And so,

(2 x+ 6)dx
∫ x 2+xdx 1
= ∫ 2
6 x +13 2 x +6 x +13
+(−3)∫ 2
dx
x +6 x+ 13

1 dx
¿ ln |x 2+ 6 x+13|−3 ∫
2 2
( x +3) +(2)
2

xdx 1 3 x +3
∫ x 2+ 6 x +13 = 2 ln ( x 2 +6 x +13 )− 2 arctan 2
+C answer

( 4 x +5 ) dx
2. ∫ x 2+ 4 x +20
Solution:
4 x+5 A (2 x + 4) B
Let 2
= 2 + 2
x + 4 x+ 20 x + 4 x+20 x + 4 x +20

∴ 4 x +5= A ( 2 x +4 ) +B

If x=−2 ,−3=B ∴ B=−3

If x=0 , 5=4 A−3 ∴ A=2

And so,

( 4 x +5 ) dx ( 2 x + 4 ) dx
∫ x 2+ 4 x +20 =2∫ x 2 +4 x +20 −3 ∫ x 2+ 4dxx +20
dx
¿ 2 ln ( x 2+ 4 x +20 )−3∫
(x +2)2+( 4)2

( 4 x +5 ) dx
∫ x 2+ 4 x +20 =2 ln ( x 2+ 4 x +20 ) − 34 arctan x +2
4
+C answer
5 WALLI’S FORMULA

5.1. Definite Integrals

Now that we have acquired some facility in integration, it is feasible to start seriously on
the many applications of the definite integral. Before taking up the applications, let us review
the techniques of definite integration.
π
3
Example . Evaluate
∫ x sin x dx .
0
We employ integration by parts to obtain,
π π
2 π 3
3
∫ x sin x dx= [−x cos x ] +∫ cos x dx 0
0 0
π
π π 3
¿− cos +0+ [ sin x ] 0
3 3
π π
¿− +sin −0
6 3
√ 3 π
¿ − =0.342
2 6

5.2. Change of Limits with Change of Variable

b
In the definite integral ∫ f ( x ) dx it is always implied that a and b are
a
the limiting values of the varaible of integration x . If we change the variable by a substitution
x=φ ( z ) Equation1
we must either return to the original variable before substituting the limits, or
change the limits ¿ correspod with the change of variable .The latter method is usually
preferable. The new limits are found, of course, from the equation of substitution Equation1.
a
x 3 dx
Example . Evaluate ∫ 5 .
0 2 2 2
(a + x )
1
Put x=a tan φ . Then dx=a sec 2 φ dφ ; when x=0 , φ=0, and when x=a , φ= π . Thus
4
we proceed as follows:
π
a 3 4
x dx a2 tan 3 φ a sec 2 φ dφ
∫ 5
=∫ 5
0 2 2 2 0
(a + x ) ( a2 sec 2 φ ) 2
π π
4 3 4
1 tan φ dφ 1
¿ ∫ 3
= ∫ sin3 φ dφ
a 0 sec φ a 0
π
4
1
¿
a0
∫ sin φ ( 1−cos2 φ ) dφ
π
1 cos 3 φ 4
[
¿ −cos φ+
a 3 0 ]
1 −1 1 1
¿
[ +
a √ 2 6 √2
− −1+
3 ( )]
1 2 5 4 2−5
= √
¿
[ −
a 3 6√2 ]
6 a √2

5.3. limitations on Certain Formulas

du u
To verify ∫ =arcsin + C , the work is as follows:
2
√ a −u 2 a
1
a2
= ∙ √2 2
d u a
1 1 1
arcsin = = ∙
du a u 2 a a2−u 2 a √ a −u
1− 2
a√ a
2

This proves the formula for the case a> 0; but if a< 0, then √ a2=−a, and
the formula must be changed to read
du u
∫ 2 2 =−arcsin a + C a< 0 Equation 1
√ a −u
The above is typical of a phenomenon that occurs many times in integration. A formula,
valid within certain ranges, is incorrect in other ranges, even though all the functions occurring
are well defined there. The commonest region of failure is for negative values of the variable of
integration x , or of some constant.
du u
With the limitation a> 0 on ∫ =arcsin + C , the standard formulas are valid
2
√ a −u 2 a
wherever the functions are defined.
2 x 2−1
Example . Find the are in the second quadrant bounded by the curve y = , the
x2
x−axis , and the line x=−2.
In the second quadrant
2
y= √
x −1
,
x
−1 −1
x 2−1
A=∫ ydx=−∫ √ dx . Equation 2
−2 −2 x
An attempt to evaluate the above integral by using the result
2 2
∫ √ x x−a a
dx =√ x 2−a2+ a arcsin +C Equation 3
x
is doomed to failure, because Equation3 is based on the assumption that x is positive. [In the
−1
1
2
[
derivation of Equation 3 , √ x is replaced by x .] Indeed, − √ x −1+arcsin
2
x −2 ]
[
¿−arcsin (−1 ) − −√ 3−arcsin
−1
2 ( )]
π
¿ √ 3+ =1.73+1.05=2.78 .
3
1
But the area A , shaded in the Figure, is less than √ 3=0.87 .
2
Figure 5. 1

Evaluation of the integral in Equation3 for negative x , replacing √ x 2 by (−x ) in the


derivation, yields
√ x2 −a2 a
∫ dx =√ x 2−a2−a arcsin + C ; x ≤−a<0. Equation 4
x x
The integral in Equation2 may now be evaluated with the aid of Equation 4, which
produces the correct answer,
π
A=√ 3− =0.68 .
3
Although, of course, such situations are not the most usual thing, they occur far too often
to be considered freakish. But it would be wearisome and time-consuming to keep constant
track of such matters in our daily work: thus, we have perforce ignored them and must continue
to do so. We leave the subject with the following injunction (applying not so much to the work
of this course, where these difficulties will be largely avoided, as to activities in applied
mathematics in which the student may now or subsequently be interested):
In an integration involving a square root or other many-valued function, particularly
when some of the quantities are negative, watch every detail closely to make sure that in each
transformation the right branch is taken.

5.4. Walli’s Formula

The integral
π
2

∫ sin m x cos n x dx , Equation1


0
in which m and n are integers ≥ 0, arises over and over again in elementary applications.
Fortunately, the integral Equation1 can be evaluated simply, with a formula which is easy to
remember in words, though bulky looking in symbols.
We shall prove that if m and n are integers ¿ 1,
π
2

∫ sin m x cos n x dx , Equation2


0
2 2

¿
[ ][
1 ]
( m−1 ) ( m−3 ) ⋯ ¿ ( n−1 )( n−3 ) ⋯ ¿
1
∙α ,
2
( m+ n ) ( m+ n−2 ) ⋯ ¿
1
in which
π
α = , if m∧n are both even ,
2
α =1 otherwise .
A∙B
In words, the value of integral Equation1 is ∙ α , in which
C
A=¿ the product, starting with one less than the exponent m , going down 2 at a time,
until 2 or 1 is reached;
B=¿ a similar product, starting with one less than the other exponent;
C=¿ a similar product, starting with the sum of the exponent.

If either m or n is unity, the integral Equation1 can be evaluated at once by the power
formula. If either m or n is zero, the result is not so simple, but one added device permits us to
include that result in the formula Equation2.
RULE. If the first factor in any of the products to be formed in applying Walli's formula, for
m , n≥ 0 ,is less than one, replace that product by unity. Example ( a ) .Evaluate
π
2

∫ sin 8 x cos 4 x dx .
0
By Walli’s formula, we obtain
π
2
( 7 ∙ 5∙ 3 ∙1 ) ( 3 ∙1 )
∫ sin 8 x cos 4 x dx= 12∙ 10 ∙8 ∙ 6 ∙ 4 ∙ 2 ∙ π2 = 7211π = 2048

.
0
π
2
Example ( b ) . Evaluate
∫ sin 5 β cos6 β dβ .
0
By Walli’s formula,
π
2 3
( 4 ∙ 2 )( 5 ∙3 ∙ 1 )
∫ sin 5 β cos6 β dβ= 11∙ 9 ∙7 ∙ 5 ∙3 ∙ 1 ∙ 1= 11∙29 ∙ 7 = 693
8
0
π
2
Example ( c ) . Evaluate
∫ cos 3 φ sin5 φ dφ .
0
At once,
π
2
(2) (4 ∙ 2)
∫ cos 3 φ sin5 φ dφ= 8 ∙ 6 ∙ 4 ∙ 2 ∙1= 81∙ 3 = 24
1
.
0
π
2
Example(d). Evaluate ∫ cos 7 φ sin φ dφ .
0
Here, in forming the product associated with the exponent of the sine, we would
normally start with one less than one, namely, with zero. Hence, by the rule above, we replace
that product by unity and write
π
2
( 6 ∙ 4 ∙ 2) ( 1)
∫ cos 7 φ sin φ dφ= 8 ∙ 6 ∙ 4 ∙ 2 = 18 ,
0
a result readily verified by direct integration. Since this integration is so easily performed by the
power formula, the student should realize that the example is included only because it permits
such simple verification. In practice we do not use Walli’s formula when one of the exponents is
unity; such as a procedure would be somewhat like using an atom bomb to remove a tree
stump.
π
2
Example ( e ). Evaluate
∫ sin 6 y dy .
0
Here one exponent is zero. Now 6+ 0=6 , to start the denominator product. Also, 6 and
10 are both even. Hence,
π
2
( 5 ∙3 ∙ 1 ) (1 ) π 5 π 5 π
∫ sin 6 y dy= ∙ =
6 ∙ 4 ∙2 2 26 32
=
0

5.5. Derivation of Walli’s Formula

First, consider the integral


π
2

T =∫ cos n x dx Equation1
0
Use the integration by part, with u=cosn−1 x , dv=cos x dx , to obtain
π
π 2

T =[ cos n−1 x sin x ] + ( n−1 )∫ cos n−2 x sin2 x dx


2
0
0
π
2

¿ 0+ ( n−1 )∫ cosn−2 x ( 1−cos2 x ) dx


0
π
2

¿ ( n−1 )∫ cos n−2 x dx ( n−1 ) T ,


0
from which
π
2
n−1
T=
n 0
∫ cosn−2 x dx Equation 2
In a like manner, replacing n by ( n−2 ) in Equation2, we find that

π π
2 2

∫ cos n−2 x dx= n−3


n−2
∫ cos n−4 x dx ,
0 0
and the process can be iterated, beating down the exponent of the cosine two at a time, until the
exponent is one or zero.
Thus, if n is even,
π
2
n−1 n−3 3 1
T= ∙ ⋯ ∙ ∫ cos0 x dx
n n−2 4 2 0
( n−1 ) ( n−3 ) ⋯ 3∙ 1 π
¿ ∙ ,
n ( n−2 ) ⋯ 4 ∙ 2 2
as described in the rule of the preceding section.
If n is odd, iteration of Equation2 yields
π
2
n−1 n−3 4 2
T= ∙ ⋯ ∙ ∫ cos x dx
n n−2 5 3 0
( n−1 ) ( n−3 ) ⋯ 4 ∙ 2
¿ ∙1,
n ( n−2 ) ⋯ 5 ∙ 3
also as described in the rule of the preceding section.
π
2 1
In order to evaluate
∫ sin n x dx, put x= 3 π− y, and thus obtain
0
π π
2 0 2
n
∫ sin x dx=−∫ cos n y dy =∫ cosn y dy ,
0 π 0
2
the integral already treated above.
Finally, consider
π
2

W =∫ sinm x cos n x dx Equation 3


0
Use integration by parts, with u=sin m−1 x , dv=cos n x sin x dx, to find that
π
π 2

W=
−1
[ sinm−1 x cos n+1 x ] + m−1 ∫ sin m−2 x cosn+ 2 x dx
2
0
n+ 1 n+1 0
π
2
m−1
¿ 0+ ∫ sin m−2 x cosn x ( 1+sin 2 x ) dx ,
n+1 0
or
π
2
m−1 m−1
W= ∫ sin m−2 x cos n x dx− W.
n+ 1 0 n+1
This last equation is easily solved for W , yielding
π
2
m−1
W= ∫ sinm−2 x cos n x dx Equation 4
m+n 0
Formula Equation 4 can be used to reduce the exponent on the sine two at a time, until
that exponent is one or zero.
If m is odd in Equation3 , then iteration of Equation 4 gives
π
2
( m−1 ) ( m−3 ) ⋯ 4 ∙ 2
W= ∫ sin x cos n x dx
( m+n )( n−2 ) ⋯ ( n+5 ) ( n+3 ) 0
( m−1 )( m−3 ) ⋯ 4 ∙ 2
¿
( m+ n )( n−2 ) ⋯ ( n+5 )( n+3 )( n+ 1 )
2
[
from which the result stated in Walli’s formula follows by inserting the factors ( n−1 ) ( n−3 ) ⋯ ¿

in numerator and denominator.


1 ]
If m is even in Equation 3 , then iteration in Equation 4 gives
π
2
( m−1 ) ( m−3 ) ⋯ 3 ∙1
W= ∫ cos n x dx
( m+n )( m+n−2 ) ⋯ ( n+4 ) ( n+2 ) 0
As the last step in obtaining the desired expression for W , we insert the value of the
integral T , of Equation 1, as determined at the beginning of this section, and we thus arrive at
Walli’s formula.

Solved Problems:
π
2
1.
∫ sin 4 β dβ
0

Solution:
π
2
( 4−1 )( 4−3 ) π
∫ sin 4 β dβ= 4 ( 4−2 )

2
0
3 ∙1 π
¿ ∙
4∙2 2
π
2

∫ sin 4 β dβ= 316π answer


0

0
x 2 dx
2. ∫ 3
−1 3 2
( 1−x )
Solution:
0 0 −3
x 2 dx −1
∫ 3
= ∫ ( 1−x 3 ) 2 (−3 x 2 ) dx
3 −1
−1
( 1−x 3 ) 2
−1 0

[ ]
3 2
1 ( 1−x )
¿−
3 1
2 −1

−1 −1
¿
2
3
[
(−1 ) 2 −( 2 ) 2
]
1 1 2 2−1
= √
¿−
3 [
1−
√2 3 √2 ]
0
x 2 dx 2−√ 2
∫ 3
= answer
−1 3 2
3
( 1−x )
6 PLANE AREAS: IMPROPER INTEGRALS

6.1. Plane Areas

In our first attack on the problem of plane area, we were greatly handicapped by limited
facility in integration. We therefore return briefly to this topic.
Example . Find the area of the loop of the curve
y 2=4 x 2 ( 1−x ) .
We have
1 1
A=2∫ ydx =4 ∫ x √1−x dx .
0 0
Integrate by parts, with u=x:
1 1 1 1
A=
−8
3
[ ] 8
x ( 1−x ) 2 0 + ∫ ( 1−x ) 2 dx
3 0
1 1
¿ 0− [ ( 1−x ) ] =
16 16 2
0
15 15
Figure 6.1
Let us obtain a check by evaluating the integral in another way. Put √ 1−x=r . Then
x=1−v 2 , dx=−2 v dx
When x=0 , r =1, and x=1 , v=0.
Therefore
0 0
A=4 ∫ ( 1−v 2 ) r (−2 vdv )=8 ∫ r 2 ( 1−r 2) dv
1 1
2 5 1
r r
¿8 −[
3 5 0 ]
1 1 16
(
¿8 − =
3 5 15 )
6.2. Substitution Suggested by the Problem

When finding plane areas by the formula


b
A=∫ ydx Equation1
a
We invariably substituted for y . But it is equally proper, and frequently more
convenient, to substitute for dx∧change ¿ y −limits. That is, we take, as the substitution
formula, the equation of thecurve itself .
Of course, similar remarks will apply in all the other applications that we shall take up.
Example(a). Find, in two ways, the area in the first quadrant bounded by the cubic
y=3 x−x 2, and the lines x=0 , y=2 . The curve is shown in the Figure 6. 2.
FIRST METHOD. Using the vertical element (not shown), we find that

1
A=∫ ( 2− y ) dx
0
1
¿ ∫ ( 2−3 x + x 3 ) dx
0
1
3 x2 x4
[
¿ 2 x−
2
+
4 0 ]
3 1 3
¿ 2− + =
2 4 4

Figure 6.2

Figure 6.2

SECOND METHOD. As a check, we find the same area, using the horizontal element
shown in the Figure 6. 2:
2
A=∫ xdy .
0
It is not feasible to substitute for x , but we may easily substitute for dy and change
limits:
dy =( 3−3 x 2 ) dx ; x=0 when y =0 , x =1 when y=2 ;
2 2
A=∫ xdy=3∫ ( x−x 3 ) dx
0 0
1
3 2 3 4 3
¿ [ 2
x− x =
4 0 4 ]
To evaluate an integral such as Equation1 when x and y are given in terms of a
parameter, we substitute for both y∧dx, taking as new limits the values of the parameter
corresponding to the given limits.
Example ( b ) . Find the area of the ellipse( Figure 6. 3)
x=a cos φ , y=b sin φ
At once
a
A=4 ∫ ydx
0

Figure 6. SEQ Figure ¿ ARABIC 3


1
When x=0 , φ= π , and when x=a ,φ=0. Therefore
2
1
π
2

A=4 ∫ ( b sin φ )(−a sin φ dφ )


0
1
π
2

¿ 4 ab ∫ sin 2 φ dφ
0
1 π
¿ 4 ab ∙ ∙ =πab
2 2
The transformation suggested in this section are intuitively reasonable. Rigorous
justification of them belongs to a course in advance calculus.

Solved Problems:
1. Find the area bounded by the curve a 2 y=x 3, the x -axis, and the line x=2 a. Solve in
two ways.

Solution 1:
2a
A=∫ ydx
0

2a
x3
¿ ∫ 2 dx
0 a
2a
1 x4
¿ 2
a 4 [ ] 0

1
¿ 2
( 2 a )4
4a

A=4 a2 sq . units answer Figure 6.4

Solution 2:

dA=( 2 a−x ) dy
8a 2 1
A=∫ (2 a−x) dy but x=a 3 y 3
0

Thus,
8a 2 1
3 3
A=∫ (2 a−a y )dy
0

2 4 8a Figure 6.5

[
¿ 2 ay−
a y
4
3
3 3

]
0

2 4 8a

[ 3
¿ 2 ay− a 3 y 3
4 ] 0

2 4
3 3
¿ 2 a ( 8 a )− a 3 ( 8 a )
4

3
¿ 16 a2 − ( 16 a2 )
4

1
¿ ( 16 a2 )
4

A=4 a2 sq . units answer

2. Find the area bounded by the curve y 2−3 x +3=0 and the line x=4.

Solution:
4
dA=2 ydx A=2∫ ydx
1

4 1
A=2∫ ( 3 x−3 )2 dx
1
4 1
2
¿
31
∫ ( 3 x−3 ) 2 3 dx
3 4

¿
3
[ ]
2 ( 3 x−3 )
3
2
2

3
4 2
¿ (9)
9

A=12 sq . units answer

6.3. Plane Areas in Polar Coordinates

Given the equation


r =f (θ)
of a plane curve in polar coordinates, let us try to find the area bounded by the curve and two
fixed radius vectors θ=α , θ= β. ( Figure 6.7)
Inscribe in the area n circular sectors of radius r i and angle ∆ θ . By elementary geometry,
1 2
the area of each sector is r ∆ θ.
2 i
n
Now add up the areas of all the sectors: ∑ 12 r 2i ∆ θ. As n increases and the sectors
i=1
become narrower and narrower, this um approaches as its limit the area under the curve . Hence,
by Theorem 19,
n β
1 1
A=lim ∑ r 2i ∆θ= ∫ r 2 dθ
n →∞ i=1 2 2α
For the present, we must rely upon geometric intuition to assure us that the limit of this
sum (i.e., the area) is the same as the one appearing in the definition.

Figure 6.7 Figure 6.8

Example . Find the area within the curve r 2=a2 cos θ .( Figure 6.8)
1
As θ varies from 0 to π , we get positive and negative values of r , giving rise to the arcs
2
1 3 3
APO , A ' P ' O . For π <θ< π ,r is imaginary. For π <θ<2 π ,we get the arcs OQA , OQ ' A ' .
2 2 2
Integrate through the first quadrant and multiply by 4:
1 1
π π
2 2 1
1 1 π
A=4 ∙ ∫ r 2 dθ=2 a2 ∫ cos θ dθ= a2 [ sin θ ]02 =0
2 0 0 2
This result, puzzling at first, is due to a peculiarity of the polar coordinate system.
Although the curve appears in the second and third quadrants, these arcs, as notes above,
correspond not to values of θ in those quadrants but to values of θ in the first and fourth
1 3
quadrants with negative r ; when π <θ< π ,r 2 becomes negative and r imaginary. When we
2 2
1 2
integrate across a region in which this occurs, each of the elements π r dθ is negative, and the
2
same is true of the limit of their sum. This illustrates the fact that in polar coordinates
it isnot safe ¿ choose thelimits merely ¿ the appearance of the curve .Here even more than in
rectangular coordinates, it is best to keep thelimits of integration as narrow as possible by using
considerations of symmetry to the fullest extent.

Solved Problems:
1
1. Find the area under one arch of the curve y=cos x.
4

Solution:

1
dA= ydx =cos x dx
4

A=2∫ dA
0


1
¿ 2 ∫ cos x dx
0 4

1 1 Figure 6.9
¿ 8 ∫ cos x dx
0 4 4

1
[
¿ 8 sin x
4 ] 0

π
(
¿ 8 sin −sin 0
2 )
¿ 8 ( 1−0 )

A=8 sq .units answer

x
2. Find the area under the arch of the curve y=a sin
a

Solution:

πa
x
∴ A=∫ a sin dx
0 a
πa
¿a 2
∫ sin xa dxa
0

Figure 6.10
πa
x
[ ]
¿−a2 cos
a 0

¿−a2 [ −1−1 ]

A=2 a2 answer

6.4. Integrable Functions

A function f ( x ) is said to be integrable in the interval a ≤ x ≤ b if the definite integral


b
A=∫ f ( x ) dx
a
exists.
We know that area
“under” any continuous curve exists; hence
every continuous function is integrable. Whether or not we can express the integral in
elementary terms in immaterial: if not, it will still be possible by more advanced methods to
evaluate it in the strict sense of the term – i. e ., to find its value, to any degree of
approximation, for given values of a and b .
Furthermore, if the function is continuous except for a finite number of finite
discontinuities in the interval as in the Figure 6 .11, the area still exists, and the function is
integrable.

Figure SEQ Figure ¿ ARABIC 6 .11

DEFINITION. If f ( x ) is continuous ∈the interval a ≤ x <b , then


b

∫ f ( x ) dx= lim
c
¿
a −¿
c →b ∫ f (x )dx ;¿
a

if f ( x ) is continuous∈theinterval a< x ≤ b ,then


b

∫ f ( x ) dx= lim
b
¿
a +¿
c→a ∫ f ( x ) dx .¿
c

Applying this definition, repeatedly, if necessary, we integrate over the separate


segments and add the results. In this connection, missing point discontinuities may be ignored,
since the value of the function, or lack of any value, at a single point cannot affect the value of
the area.
Example(a). Find the area in the first quadrant under the curve
y=1+ x 2 , x ≤1
¿ 2−x 2 , x >1
With the aid of Figure 6.12 we write
√2
A=∫ ydx
0

Figure 6.12
1
¿ ∫ ( 1+ x 2 ) dx
0
+ lim ¿
√2
c →1
+¿
∫ (2− x2 ) dx¿
0

Since ( 2−x2 ) is continuous at x=1, we have at once


1 √2
A=∫ ( 1+ x 2) dx +∫ ( 2−x2 ) dx
0 0
1
Simple integrations lead quickly to the evaluation A= ( 4 √2−1 )=1.55.
3
Example ( b ) . There occurs the function
1 1

d
√√ √ √
f ( x )= x + x− + x− x−
4 4
1
Let y= [ xf (x ) ] . Find the area under the y−curve from x= to x=1. See Figure 6.13.
dx 4
In this example the function y (x ) is chosen as the derivative of a known function in
order to make available a simple check on our answer.
1
It can be shown that f ( x ) is imaginary for x < and
4
1 1
≤x ≤ , f ( x )=1
4 2
1
¿ √ 4 x−1 x >
2
d
Therefore, since y= [ x ∫ ( x ) ],
dx
1 1
y=1 ≤ x ≤
4 2
6 x−1 1 Figure 6.13
¿ x>
√ 4 x−1 2
The desired area may now be obtained from
1 1 1
( 6 x−1 ) dx
A=∫ y dx=∫ 1∙ dx +∫
1 1 1 √ 4 x−1
2 2 2
3 1
1 ( 4 x−1 )+
1 2 2
¿ +∫ dx
4 1 √ 4 x−1
2
or
1 1 1 −1
1 3 1
A= + ∫ ( 4 x −1 ) 2 dx + ∫ ( 4 x−1 ) 2
dx
4 21 21
2 2
1
It is then a simple matter to show that A=√3− .
4
As a check, note that
1
A=∫ y dx
1
2
¿ lim
k
¿
1−¿
+¿ ,k→
2 c
∫[ xf (x ) ]'dx+ lim ¿¿
1
1
+¿ '
a→ 1 ∫ [ xf ( x) ] dx ¿

c→ 2
a
¿
2
¿ lim ¿
1
1 +¿ ,k→ 2
c→ [ xf ( x ) ] kc + lim ¿¿
2 1+ ¿ 1
a→ [xf ( x )]a ¿
2

¿ lim ¿
−¿
1
k→ kf (k)− lim ¿¿
2 c→
1 +¿
cf (c )+ f (1 )− lim ¿¿
2 a→
1+ ¿
af ( a ) ¿
2

Hence
1 1 1 1
A= ∙ 1− ∙ 1+ √ 3− ∙ 1=√ 3−
2 4 2 4

6.5. Improper Integrals

In certain instances, a meaning may be assigned to the function


b
A=∫ f ( x ) dx
a
under either or both of the following circumstances:
(a) Either a or b , or both, increase numerically without bound; or
(b) The integrand f (x) has an infinite discontinuity at an endpoint or one more
interior points of the interval.

In either case, the integral is called an improper integral .

6.6. Integrals with Infinite Limits

If we keep a fixed, the integral


b
A=∫ f ( x ) dx
a
becomes a function of b only. It may happen that as b increases without bound, the function A

approaches a limit. If so, this limit is denoted by the symbol ∫ f ( x )dx :
a
∞ b

∫ f ( x )dx= blim
→∞
∫ f ( x )dx
a a
Similarly,
b b

∫ f (x) dx= lim a→−∞ a


∫ f (x )dx ;
−∞
∞ c b

∫ f (x) dx= a→−∞


lim ∫ f ( x )dx + lim ∫ f (x) dx Equation 1
b→∞
−∞ a c
where c may have any fixed value. If the limits occurring in the right members do not exist, the
integrals on the left have no meaning.
An improper integral for which the defining limit or limits do not exist is said to be
divergent . If the defining limits exits, the integral is called convergent . The terms convergent
and divergent occur again in the study of infinite series with meanings similar to those used
here. We make little use of the term convergent in connection with improper integrals; for such
an integral we obtain its value and therefore have no need for a term which expresses merely
the fact that it does have a value. The term convergent is employed widely in advanced calculus,
which contains a study of tests to determine whether an improper integral exists, without
necessarily evaluating the integral.
∞ b
It should be noted that ∫ f (x)dx does not mean blim
→∞
∫ f ( x ) dx.
−∞ −b
If the former exists, then the latter also exists, and the two limits are equal; but the latter may
exist when the former does not. For example,
b b
x2 b2 b 2


lim ∫ x dx=lim
b → ∞ −b b→ ∞ 2
[ ] =lim
−b b → ∞ 2
(
− =0 ;
2 )
but ∫ x dx is divergent, since neither limit in Equation1 exists.
−∞
∞ b b
dx dx
Example . ∫ 2 =lim ∫ 2 = lim
1 x b→∞ 1 x b→ ∞
−1
x 1
=1. [ ]
1
The curve y= 2 is show in Figure 6.14 . Geometrically, the above integral means the
x
limit of the shaded area as b becomes infinite. This limit we define as the “area bounded by” the
curve, the x -axis, and the line x=1, although it is not properly a bounded area in the literal
sense. It is evident that a similar argument holds in general: An integral with an infinite

Figure 6.14
limit may be interpreted as the are under a curve which approaches the x−axis , usually
without ever reaching it. In ordinary cases, the x−axis is an asymptote of the curve. However,
even though the curve is asymptotic to O x, the integral is not necessarily convergent.

Solved Problems:
Find the area enclosed by the given curve
1. r =2 a cos θ

Solution:

1
dA= r 2 dθ
2
π
1
A=∫ (2 a cos θ )2 dθ
0 2

π
Figure 6.15
¿ 2 a2∫ cos2 θ dθ
0

π
¿2a 2
∫ 1+cos
2


0

π
1
[
¿ a2 θ+ sin 2 θ
2 ]
0

¿ a2 ( π )

A=π a 2 sq . units answer


2. r =2 a cos2 θ

Solution:

r =2 a cos2 θ

1
dA= r 2 dθ
2
π
2
1 2
A=4 ∫ ( 2 a cos 2 θ ) dθ
0 2

π
2

¿ 8 a2∫ cos 4 θ dθ
0
Figure 6.16

By Walli’s Formula:

( 4−1 )( 4−3 ) π
A=8 a2 ∙
( 4 ) ( 4−2 ) 2

(3) ( 1) π
¿ 8 a2 ∙
( 4 )( 2 ) 2

3π 2
A= a sq . units answer
2

6.7. Infinite Discontinuities of the Integrand

Consider now the second class of improper integrals in


which the limits area finite, but the integrand has an infinite
discontinuity at an endpoint or an interior point of the interval.
DEFINITIONS:
If f ( x ) increases numerically without bound as x → b−¿,¿
b

∫ f ( x ) dx= lim
c
¿
a −¿
c →b ∫ f (x )dx ;¿
a

Figure 6. SEQ Figure 6. ¿ ARABIC 1 7


if f ( x ) increases numerically without bound as x → a+ ¿,¿
b

∫ f ( x ) dx= lim
b
¿
a +¿
c→a ∫ f ( x ) dx.¿
c

Example . Find the area bounded by the


curve x y 2 =1, the axes, and the line x=1.
We write
1
A=∫ y dx= lim ¿
1
0 dx +¿
c→ 0 ∫ ¿
c √x
¿ lim ¿
1
c →0+ ¿ [ 2 √ x ] c = lim ¿¿
+¿
c→0 ( 2−2 √ c )¿

¿2

Solved Problems:
Evaluate the integrals.

1. ∫ dx
x
3
2

Solution:
∞ b

∫ dx3
dx
=lim ∫ 3
2 x b→∞ 2 x

b
¿ lim ∫ x−3 dx
b→∞ 2

b
x−2
¿ lim [ ]
b → ∞ −2 2

−1 1
¿ lim
b→∞ [ ] 2
+
2 b 2 ( 2)
2

∫ dx
3
x 8
1
= answer
2


2. ∫ e−2 y dy
0

Solution:
∞ b
−2 y
∫e dy= lim ∫ e−2 y dy
0 b→∞ 2

−2 y b
¿ lim
b→∞
( −12 ) [ e ] 0

−1 1
2 [e ]
¿ lim ( ) −1 −2 b
b→∞

∫ e−2 y dy= 12 answer


0
7 APPLICATION OF INTEGRATIONS

7.1 The General Method

Theorem 19 states that if f(x) is continuous in the integral a ≤ x ≤ b ,


if a=x 1< x 2 < x 3 <⋯< x n ≤ x n +1=b ,
and if ∆ x k =x k +1−x k , for k =1,2,3 ⋯ , n , then
n b
lim ∑ f ( x k) ∆ x k =∫ f ( x ) dx .
max . ∆ xk → 0 k=1 a
It might be thought that the theorem applies only in the computation of plane areas. But
any function of one variable may be represented graphically as a plane curve. Thus, Theorem 19
may be used to evaluate,
n
lim ∑ f ( xi ) ∆ x
∆ x→ 0 i=1
Regardless of the physical meaning of the function f(x).

7.2 Solids of Revolution: Circular Disks

Let a solid be generated by rotating the area OAB about the x-axis. Imagine this solid cut
into thin slices by planes perpendicular to the axis of revolution (a typical slice being formed by
the rotation of the area PQRS). Trim off the irregular outer edge that is generated by revolving
the area S’RS, to leave a thin circular disk that is generated by revolving the rectangle PQRS’.
2
The radius of this element is y, the thickness ∆ x , the volume πy i ∆ x . Now, as the disks are taken
thinner and thinner, the aggregate volume of trimmings approaches zero, and the sum of all the
elementary volume πy 2i ∆ x approaches as its limit the volume of the solid:
n b
V = lim ∑ f ( x i ) ∆ x=π ∫ y 2 dx
∆x →0 i=1 a

Figure 7. SEQ Figure7. ¿ ARABIC 1

Example. The area bounded by a parabola, its axis, and its latus rectum revolves
about the axis. Find the volume generated.
Let the equation of the parabola be y 2=4 ax . Dividing the area into elements, we
see that each rectangle generates a cylindrical volume-element of radius y, altitude dx,
and volume πy 2 dx . Hence,
a a
V =π ∫ y dx=4 πa ∫ xdx=2 πa2
2

0 0
Figure 7. SEQ Figure7. ¿ ARABIC 2

Example. The above area rotates about the latus rectum. Find the volume
generated.

Figure 7. SEQ Figure7. ¿ ARABIC 3


2a
V =π ∫ ( a−x )2 dy
0
2a 2
y2
¿ π∫
0
( a−
4a )
dy
2a 2
y2 y4
¿ π ∫ a−
0
( +
2 16 a2
dy)
16
V= π a2
15

7.3 Solids of Revolution: Circular Rings

In the general argument, the axis of rotation formed of the boundary of rotating area;
but the method works equally well when this is not the case. If the area is to revolve about the
x-axis, we may use as element the circular ring, or washer, formed by revolution of the rectangle
PQRS.
Figure 7. SEQ Figure7. ¿ ARABIC 4

Example. Find the volume generated by revolving a circle about one of its tangents.

Figure 7. SEQ Figure7. ¿ ARABIC 5

Let the circle,


x 2+ y 2=a2
revolve about the line x=a . The volume-element is a circular ring of outer radius S P' =a+ x ,
inner radius P P ' =a−¿, thickness dy :
a
V =π ∫ [ ( a+ x )2−( a−x )2 ] dy
−a
which reduces to
a
V =8 πa ∫ xdy
0
Since this integral represents the area of the circular quadrant, we know its value:
1 2
V =8 πa ∙ π a =2 π 2 a3
4
Solved Problems:
1. The area bounded by the curve y=e−x , the axes, and the line x=2 is revolved about
the x−axis . Find the volume generated.

Solution:

dV =π y 2 dx
2
∴ V =π ∫ y 2 dx
0

2
¿ π ∫ e−x dx
0
2
π
¿− ∫ e−2 x (−2 dx )
2 0

π −2 x 2
¿− [ e ]0
2

π
¿− ( e−4−1 )
2

π 1
¿
2 (
1− 4
e )
Figure 7. 1
1
V = π ( 1−e−4 ) cu .units answer
2

2. The area under one arch of the sine curve revolves about the x−axis . Find the volume
generated.

Solution:

dV =π y 2 dx
π
V =π ∫ sin2 x dx
0

π
1−cos 2 x
¿ π∫ dx
0 2
π
1 1
¿π [ 2
θ− sin 2 x
4 ] 0
Figure 7. SEQ Figure7. ¿ ARABIC 7
¿π ( 12 π)
π2
V= cu . units answer
2

7.4 Solids of Revolution: Cylindrical Shells

Let a solid be formed by revolving the area OAB about Ox. Divide the solid into thin
shells, each with its axis in the axis of revolution; a typical shell is formed by rotation of the strip
PQRS. Trim the outer end, leaving the cylindrical shell formed by rotation of the rectangle PQRS'
. the inner radius is y i +∆ y , height x i, volume
∆ V i =π ( y i + ∆ y )2 x i−πy 2i x i=2 πyi x i ∆ y+ πx i ( ∆ y )2 Equation1
Figure 7. SEQ Figure7. ¿ ARABIC 8

Hence, the volume of the solid is


n
2
V = lim
∆ y →0
∑ [ 2 πy i x i ∆ y + πxi ( ∆ y ) ]
i =1
n n
2
¿ lim ∑ 2 πy i xi ∆ y + lim ∑ πx i ( ∆ y )
∆ y →0 i=1 ∆ y→ 0 i=1
Now, in regard to the second summation, we may write,
n n
2
lim ∑ πxi ( ∆ y ) =π lim ∑ x i ∆ y ∙ ∆lim ∆ y=0
∆ y→ 0 i=1 ∆ y → 0 i=1 y →0
since
n
lim ∑ x i ∆ y= A
∆ y→ 0 i=1
where A is the rotating area. It follows that in the expression for the volume-element, we may
write,
n b
lim ∑ 2 πy i x i ∆ y =2 π ∫ yx dy Equation 2
∆ y→ 0 i=1 a
From Equation 2, a volume of revolution may be obtained by taking the limit of the sum
of (integrating) cylindrical shell element, each of which is equal in volume to the circumference
times the height times the thickness.

Solved Problems:
1. Find the volume of a sphere.

Solution:

dV =2 πx (2 y ) dx=4 πxy dx

Since y =√ a2−x 2

Then dV =4 π √ a2−x 2 xdx

Figure 7. SEQ Figure7. ¿ ARABIC 9

a 1
2 2 2
Hence , V =4 π ∫ ( a −x ) x dx
0
a 1
2 2 2
¿−2 π ∫ ( a −x ) (−2 xdx )
0

3 a

[ ]
2 2 2
(a − x )
¿−2 π
3
2 0

4
¿− π [ 0−a3 ]
3

4
V = π a3 cu . units answer
3

2. Find the volume of a circular cone.

Solution:

dV =2 πxy dx

−h hx
but y = x +h∨ y=h−
r r

Thus,
r
hx
V =∫ 2 π h−
0
( r
xdx )
r
h x2
¿2π∫
0
( hx−
r
dx )
r Figure 7. SEQ Figure7. ¿ ARABIC 10
h x2 h x3
¿2π
2[ −
3r ] 0

1 1
¿ 2 πh r 2 [ ]

2 3

1
V = π r 2 h cu .units answer
3

7.5 Miscellaneous Solids

The volume of any solid can be expressed as a definite integral, provided we know the
area of every plane section parallel to some fixed plane. We divide the solid into thin slices by
means of n planes parallel to the fixed plane, trim off the outer edge, and take as element the
slab remaining. The volume of this slab is the thickness times the area of the face, which by
hypothesis is known.
The only plane figures whose area we are supposed to know offhand are the rectangle,

the triangle, the trapezoid, the circle, the circular sector ( 13 r a), and the ellipse ( πab ). Thus, the
2
only solids whose volumes we can find at this time are those that can be divided into parallel
slices of one of these shapes.

Solved Problems:
1. Find the volume of an ellipsoid.

Solution:

dV =πxy dz

with πxy=¿ total area of any cross-section perpendicular to the

z−axis .
2
z
but x=a 1−
√ ()
c

2
z
and y=b 1−
√ ()
c

2
z
thus, dV =πab 1−
[ ( )] c
dz

therefore,
c c
πab
V =πab ∫ dz− 2 ∫ z 2 dz Figure 7. SEQ Figure ¿ ARABIC 11
0 c 0 7.

c
πab z 3
c
¿ πab [ z ] − 2
0
c 3[ ] 0

πabc
¿ πabc−
3

2
¿ πabc cu . units
3

Since this is just half the total volume,

4
V T =2V = πabc cu . units answer
3

x2 y 2
2. Find the volume in the first octant inside the cylinder + =1, under the plane
a2 b 2

z=mx .

Solution:

1
dV = xz dy since the volume element is triangular
2
1
y2
But x=a 1−
( ) b2
2

1
y2
And z=mx=ma 1− 2
b ( ) 2

Thus,

1 y2
(
dV = ma 2 1− 2 dy
2 b )
b
1 y2
2
2
V = ma ∫ 1− 2 dy
0 b ( )
b
1 y3
2
2
[
¿ m a y− 2
3b ]
0
Figure 7. SEQ Figure7. ¿ ARABIC 12

1 b
2 [ ]
¿ m a2 b−
3

1
V = ma 2 b cu units answer
3

7.6 Length of a Curve

We defined the differential ds of arc length by


2 2
ds=√ ( dx ) + ( dy ) Equation 1
Instead of Equation 1 it is sometimes convenient to use equivalent forms
1 1
dy 2 2 dx 2 2
ds= 1+ [ ( )] [ ( )]
dx
dx= 1+
dy
dy Equation 2

or, if x and y are known in terms of parameter l , the form


1
dx 2 dy 2 2
ds= [( ) ( ) ]
dl
+
dl
dl Equation 3

In agreement with Equation1we now define the length s of a curve along an arc C by,
b
dy 2
s=∫ ds=∫ 1+
c a √
dx ( )
dx Equation 4

dy
where a and b are the abscissas of the endpoints of C , and where must be replace by its
dx
value in terms of x from the equation of the curve.
If it is more convenient to integrate with respect to y , we put
d
dx 2
s=∫
c √(dy )
+ 1dy Equation5
If x and y are given in terms of parameter t ,
t
dx 2 dy 2
2

s=∫
t
dt
+
dt 1
√(
dt Equation6 )( )
We assume that in any instance the integrand is a single-valued function of the variable
of integration. If this condition is not satisfied, C must consist of several portions for each of
which the condition holds, and each portion may be considered separately.
The equation of the curve may be known in polar, rather than in rectangular,
coordinates. From x=rcosθ , y=rsinθ, it follows that,
dr 2 2
2 2 2
dx= √ ( dr ) +r ( dθ ) = [( ) ]

+ r dθ Equation 7
Solved Problems:
1. One branch of the curve 9 y 2=4 x 3 from x=0 to x=3.

Solution:

dy 2
dS= 1+

dx ( )
dx

9 y 2=4 x 3 18 ydy=12 x 2 dx
3
2
y= x 2
3
1
dy 12 x 2 2 x 2
∴ = = =x 2
dx 18 y 3 2 32
x
3

And so, dS=√ 1+ x dx

Thus,
Figure 7. SEQ Figure7. ¿ ARABIC 13
3 1
2
S=∫ (1+ x ) dx
0

3 3

¿
[ ] ( 1+ x )
3
2
2

3 3
¿
2
3
[
( 4 ) 2 −( 1 ) 2 ]
2
¿ ( 8−1 )
3

14
S= units answer
3

π
2. Find the length of the curve y=ln cos x from x=0 to x= . Use x as the variable of
4
integration.

Solution:
−sin x dx
y=ln cos x ∴ dy= =−tan x dx
cos x

Hence,

dy 2

dS= 1+ ( )
dx
dx

2
¿ √ 1+ (−tan x ) dx

dS=sec x dx
π
4

S=∫ sec x dx
0

π
4
Figure 7. SEQ Figure7. ¿ ARABIC 14
¿ [ ln |sec x + tan x|]
0

π
sin x+ 1
¿ ln [cos x ]
0
4

[
¿ ln
sin
4
cos
+1
π
4
−ln
sin 0+1
cos 0
]
√2 +1
2 0+1
¿ ln −ln
√2 1
2

2+2
¿ ln √
√2
S=ln ( 1+ √2 ) units answer

7.7 Surfaces of Revolution

The problem for this section is to find the area of the surface generated by rotating a
plane curve C about a line in its plane.
Figure 7. SEQ Figure7. ¿ ARABIC 15

To obtain the formula, we cut the pertinent section along the axis of revolution into
equal segments ∆ x which project onto elements ∆ s, along the curve C . Next, inscribe in C a
broken line of n segments ∆ s'i . In the rotation, each segment ∆ s'i generates the frustrum of a
circular cone, the radii of whose bases are y i , y i + ∆ y i. By elementary geometry, the surface area
of this conical frustrum is the circumference of the middle section multiplied by the slant height,
1
(
or 2 π y i+
2 )
∆ y i ∆ s'i . It can be shown that,
n
1
(
lim ∑ 2 π y i+ ∆ y i ∆ s'i=0
n → ∞ i=1 2 )
Therefore,
n
A=lim ∑ 2 π y i ∆ s 'i=2 π ∫ y ds
n →∞ i=1 c
Solved Problems:
1. Find the surface area of a sphere of radius a .

Solution:

dy 2
dA=2 πx dS dS= 1+
√ ( ) dx
dx

dy −x
2 x dx +2 y dy=0 y dy=−x dx ∴ =
dx y

dy 2 x2 x2
( )
dx
= 2= 2 2
y a −x

And so,

x2

dA=2 πx 1+ 2 2 dx
a −x

a 2−x2 + x 2
¿ 2 πx
√ a2−x 2
dx

Figure 7. SEQ Figure7. ¿ ARABIC 16


a
¿ 2 πx
(√ a −x 2
2 ) dx

2 x dx
dA=πa
√a 2−x 2
Thus,
a 1
2 2 2
A=−πa∫ ( a −x ) dx
0

1 a

[ ]
2 2 2
( a −x )
¿−πa
1
2 0

¿−2 πa ( 0−a )

¿ 2 π a2

Total area, AT =2 A

AT =4 π a2 answer

2. Find the surface area of a zone cut from a sphere by two parallel planes at a distance h
apart.

Solution:

dx − y
x dx+ y dy =0 =
dy x

dx 2

dS= 1+ ( )
dy
dy

dx 2 y 2 y2
( )
dy
= 2= 2 2
x a −y

Figure 7. SEQ Figure7. ¿ ARABIC 17

dA=2 πx dS

x=√ a2 − y 2

y2
dA=2 π √ a 2− y 2 1+
√ a2− y 2
dy

¿ 2 πa dy
h
∴ A=2 πa∫ dy
0

h
¿ 2 πa [ y ] 0

A=2 πah sq .units answer


8 CENTROIDS

8.1. Density

A mass is said to be homogenous if the masses contained in any two equal volumes are
equal. In all other cases the mass is heterogenous . In the present chapter we confine our
attention to homogenous masses.
The density of a homogenous mass is the ratio of the mass M to the volume V that it
occupies. That is, the density is the mass per unit volume.
A mass point may be imagined as the limiting form approached by a body whose
dimension approach zero, while the density increases in such a way that the mass remains
finite. Similarly, we may think of masses of one dimension and two dimensions—i.e., of material
curves and surfaces. Such masses are represented approximately, for example, by slender wires
and thin sheets of metal. In these cases, we define the density as “linear density”, or mass per
unit length, and “surface density”, or mass per unit area.

8.2. First Moment of Mass

The product mass m , concentrated at a point P, by the distance l of P from a given point,
line, or plane, is called first moment of m with respect to the point, line, or plane (also called
imple moment , or mass moment ). Denoting thus moment by G , we have
G=ml
Second moments, those involving the square of the distance from P to a point, line, or
plane, will be considered in Moment of Inertia.
If a system of points P1 , P2 … , P n having masses m 1 ,m 2 , … mn , respectively, be referred
to rectangular coordinate axes, the first moments of the system with respect to the three
coordinate planes are
n n n
G ys =∑ mi x i Gsx =∑ mi y i Gxy =∑ mi z i Equation 1
i=1 i=1 i=1
If the particles all lie in one of the coordinate planes, the moments with respect to
coordinate planes reduce to moments with respect to coordinate axes.
The idea of mass moment may be extended to the case of a continuous mass by thinking
of the body as composed of an indefinitely large number of particles. The actual computation of
such a moment is usually affected by means of definite integrals; we return to this question
presently.

8.3. Centroid

Given any mass M ,let G ys , G sx , G xy denote the first moments of the mass with respect to
the coordinate planes. The point C, whose coordinates x́ , ý , ź are given by the formulas
M x́ =G ys M ý=G sx M ź=G xy Equation 1
has the property that the moment of the mass with respect to each coordinate plane is the same
as if the whole mass were concentrated at that point. For, the moments for a particle of mass M
at the point C are M x́ , M ý , M ź , and by Equation1, these are equal to the moments for the
original distribution.
It can be shown that this property holds for first moments with respect to any other
plane. For a system of mass particles, the proof is as follows. Let,
x cos α + y cos β + z cos γ = p Equation 2
be the distance of any plane in the normal form, let ṕ , p 1 , … , p n be the distance of the points
C , P1 , … , Pn from this plane. Then
p1=x 1 cos α + y 1 cos β + z 1 cos γ −p ,

pn=x n cos α + y n cos β+ z n cos γ− p
so that by simple addition
n n n n n

i=1
(
∑ mi p i= ∑ mi x i
i=1
) cos α+
(∑ )
i=1
m i y i cos β+
(∑ )
i=1
(∑ )
m i z i cos γ −
i=1
mi p

¿ M x́ cos α + M ý cos β + M ź cos γ −Mp


¿ M ( x́ cos α + ý cos β+ ź cos γ− p )
¿ M ṕ
The point C is called centroid (also called center of mass , or center of gravity ). Hence:
The centroid of a mass is a point such that the first moment of the mass with respect to
any plane is the same as if the whole mass were concentrated at that point.
The first moment of a mass with respect to any plane through the centroid is zero.
In the determination of centroids, the following considerations are often useful (the first
two apply only to homogenous masses):
(a) If the body has a geometrical center, that point is the centroid.
(b) Any plane or line of symmetry must contain the centroid.
(c) If the body consists of several portions for each of which the centroid can be
found, each portion may be imagined concentrated at its centroid for the sole
purpose of computing first moments.
In many applications, we are concerned with centroids of purely geometric figures
(volumes, areas, lines), no idea of mass beinginvolved . However, it is unnecessary to write out
separate theory for such cases; the above discussion applies to plane areas for instances, if we
merely replace the word “mass” throughout by “area”. To see this, note that for a homogenous
body of given size and shape, both the mass and its moment with respect to any plane are
proportional to the density δ . Hence, in the formulas for x́ , ý , ź, the factor δ cancels out from
both members, leaving the coordinates of the centroid independent of the density. We have
therefore that δ =1, so that the “mass” is numerically equal to the area.

8.4. Centroid of a System of Particles

Combining Equation1 of 8.2 and Equation1 of 8.3, we see that the coordinates of the
centroid of a sytem of particles are given by the formulas
n n n
M x́ =∑ mi x i , M ý=∑ mi y i , M ź=∑ mi z i
i=1 i=1 i=1
By use of (c ) in 8.3, problems involving distributed masses may frequently be reduced
to consideration of a set of particles.

Solved Problems:
1. Find the centroid of the system of equal masses at ( 2 , 0 ) , ( 1 , 1 ) , ( 5 , 2 ) ,∧(−3 ,5).

Solution:

M x́ =∑ mi x i

¿ m 1 ( 2 ) +m2 (1 )+ m3 ( 5 ) +m 4 (−3 )

M =m1 +m2+ m3 + m4

2 m 1+ m 2+ m53−3 m 4
∴ x́=
m 1 +m 2+ m3 +m 4

since m 1=m 2=m 3=m 4=m


2 m+m+5 m−3 m 5 m 5
Then x́= = =
4m 4m 4

M ý=∑ mi y i

¿ m 1 ( 0 )+ m2 ( 1 ) +m 3 ( 2 ) + M 4 (−5 )

m+2 m−5 m −2 m −1
ý= = =
4m 4m 2

∴ Centroid is at ( 54 ,− 12 ) answer
2. A slender rod 40 in. long is bent so as to form a right angle. If the segments are 8 in.
and 32 in. long, find the centroid.

Solution:

Figure 8. SEQ Figure 8. ¿ ARABIC 1

LT x́=L1 x 1 + L2 x 2

x́=¿ ¿

x́=12.8∈¿

LT ý=L1 y 1+ L2 y 2

ý=¿¿

ý=0.8∈¿

Hence,

C ( 12.8 , 0.8 ) answer

8.5. Determination of Centroid by Integration

To find the centroid of a continuous mass, we must as a rule resort to integration. In the
most general case multiple integrals must be used, but in many cases of practical importance
the result may be obtained by methods analogous to those of Chapter 7.
In the following discussion we restrict ourselves to one-, two-, or three- dimensional
bodies of the forms considered in Chapter 7. Nevertheless, the formulas obtained are applicable
with proper interpretation, to all masses. Let us choose a suitable geometrical element by ∆ mi.
Let x i , y i , z i be the coordinates of the centroid of ∆ m i. Then the product x i ∆ mi is the simple
mass moment of ∆ mi with respect to the yz -plane (or the y -axis, for a plane mass moment of
∆ mi in the xy -plane), and the limit of the sum of all such moments is the moment of the whole
mass. In this way we obtain the following formulas:
n
M x́ =lim ∑ xi ∆ mi=∫ x c dm Equation1
n →∞ i=1
n
M ý=lim ∑ y i ∆ mi=∫ y c dm Equation 2
n →∞ i=1
n
M ź =lim ∑ zi ∆ mi=∫ z c dm Equation3
n →∞ i=1

where x c , y c , z c are the coordinates of the centroid of the element .


The subscript is inserted to make sure that we never forget the meaning of the
multipliers x c , y c , z c . Of course, in any problem these variables, as well as the mass element,
must be expressed in terms of some one variables.
As usual, we have omitted writing in Equations 1−3, those terms which will drop out
the limit n → ∞. For instances, it often happens that the x−coordinates is, exactly,
1
x i+ ∆ x , so that the first limit above is
2
n
1
(
M x́ =lim ∑ x i + ∆ x ∆ mi
n →∞ i=1 2 )
But
n
1
lim
n→∞
∑ ( xi + ∆ x ) ∆ mi =∫ dm=M
i=1 2
and ∆ x →0 as n → ∞, so that
n
1
lim ∑ ∆ x ∆ mi=0
n → ∞ i=1 2

8.6. Centroid of a Plane Area: Rectangular Coordinates

In any system of coordinates, Equations 1∧2 from 8.5 become, for a plane area,
A x́=∫ x c dA A ý=∫ y c dA Equation 1
In rectangular coordinates, the area element dA is, of course, the usual rectangular
element.
THEOREM 26. The centroid of a triangular area is at the intersection of the
medians.
The proof of this theorem is a very easy problem in integration. Also, a special device
may be noted. Divide the triangle into strips parallel to the base. The centroid of each strip is in
the median, so that the same must be true for the centroid lies in one median it must lie in all
three.

Figure 8. SEQ Figure 8. ¿ ARABIC 2

Solved Problems:
1. The semicircular area inside x 2+ y 2=a2 with y ≥0 . Find the centroid of the
area.
Solution:

Figure 8. SEQ Figure 8. ¿ ARABIC 3

By symmetry, x́=0 .

For ý :

A ý=∫ ydA

¿ ∫ 2 xdy since x=√ a2 − y 2

a 1
2 2 2
A ý=∫ ( a − y ) 2 y dy
0

3 a

[ ]
2 2 2
(a − y )
¿−
3
2 0

2
¿− [ 0−a3 ]
3

2
A ý= a3
3

1 2
Since A= π a
2

Then,

2 3
a
3
ý=
1 2
πa
2

4a
ý=

Therefore,

4a
(
C 0,
3π )
answer
2
2. The area bounded by the curve y 2=4 ax , the x-axis, and the line x=x 1 A= x1 y 1 ( 3 )
Solution:

Figure 8. SEQ Figure 8. ¿ ARABIC 4

A x́=∫ xdA dA= ydx A x́=∫ xydx

1
But y=2 a x 2

Thus,

1 x1 3
2 2
A x́=∫ x 2 √ a x dx=2 √ a∫ x dx
0

5 x1

¿2 √a
[]
x
5
2
2

5
4
A x́= √ a x12
5

And so,
5
4 3
√ a x 12 √ a x 2
5 12 1
x́= = 1
2 10
x1 y1 2 √ a x 12
3

3
x́= x 1
5

For ý :
x1 x1
y 1
A ý=∫ y dx= ∫ y 2 dx
0
2 20
x1 x1
1 x2
¿ ∫ 4 ax dx=2a
20 2 [ ] 0
A ý=a x 21

Thus,

a x 21 y 21
ý= a x 1=
2 4
x y
3 1 1

y 21

¿
x1 ( )
4
2
x y
3 1 1

3
ý= y
8 1

Therefore,

C ( 35 x , 38 y ) answer
1 1

8.7. Centroid of a Plane Area: Polar Coordinates

In a definite integration, if we always make sure that the limit of the sum of elements
represents the quantity required, nothing else matters. In two different problems using the
same coordinates and dealing with the same kind of physical entity (e.g., a plane area) or even
at different stages of the same problem, we may at any time shift form one type of element to
another, according to convenience.
Figure 8.5 represents an enlargement of a single element of polar area. we may adjust
the technique of 8.6 to the use of polar coordinates once we have the location of the centroid of
the sector in Figure 8.5. Actually, we shall first obtain the centroid if the sector in Figure 3 and
then find the centroid in Figure 8.5 by rotation of axes.

Figure 8. 5 8.6the area is Figure 8. 6


In Figure A=a x 2, and we know that the centroid of the sector lies
on the line of symmetry. Hence, ú=v́ cot α . By the methods of 8.5,
r sin 2α 1

A v́ = ∫
0
[ 2 2 2
]
v ( r −v ) −v cot 2α dv Equation1
The integration to be performed in Equation1 are simple. The student should show that
r
v́= (1−cos 3 2 α −sin2 2 α cos 2 α )

r ( 1−cos 2 α ) 2 r sin 2 α
¿ =
3α 3α
Since ú=v́ cot α , we may now write the coordinates of the centroid in Figure 8.6:
2 sin α 2 sin α
ú= r cos α , v́= r sin α Equation2
3 α 3 α
1
To obtain Figure 8.6 from Figure 8.5, choose α = ∆ θ and rotate the axes through the
2
1
( )
angle θ+ ∆ θ . By the rotation formulas of analytic geometry if follows that the centroid of the
2
sector in Figure 8.5 has the coordinates
1 1
( 2 ) ( 2 )
x́=ú cos θ+ ∆ θ − v́ sin θ+ ∆θ Equation3

1 1
( 2 ) ( 2 )
ý=ú sin θ+ ∆ θ −v́ cos θ+ ∆ θ Equation 4
1
Because of Equation2 with α = ∆ θ , Equations 3∧4 lead to
2
1
sin ∆ θ
2 2
x́= r cos ( θ+ ∆ θ ) Equation5
3 1
∆θ
2
1
sin ∆ θ
2 2
ý= r sin ( θ+ ∆ θ ) Equation6
3 1
∆θ
2
Since,
1
sin ∆θ
2
lim =1 ,
∆θ→0 1
∆θ
2
we can replace the x́ and ý of Equations 5 and 6 by
2 2
x c = r cos θ y c = r sin θ Equation7
3 3
In the integrands for the determination of the centroid. Note that the same coordinates,
Equation 7, may be obtained by using the shaded triangle in Figure 8.7.
When the equations of the bounding curves are to be used in polar coordinates, we
therefore obtain the centroid ( x́ , ý ) of the area by using
β
1
A= ∫ r 2 dθ Equation 8

β
1 2
A x́= ∫ cos θ ∙ r 2 dθ
2α 3
β
1 2
A ý= ∫ sin θ ∙ r 2 dθ
2α 3
Figure 8. 7

Solved Problems:

1. The upper of the area bounded by the curve r 2=a2 cos θ . ( A=a 2)

Solution:

Figure 8. 8
π
2
1 2
A ý=2 ∙ ∫ r sin θ r 2 dθ
20 3
π
2 3
1
¿ 2 ∙ ∫ a 3 ( cos θ ) 2 sin θ dθ
30
π
3 2 3
a
¿2∙
3
∫ ( cos θ ) 2 sin θ dθ
0

π
3 2 3
2a
¿−
3
∫ ( cos θ ) 2 (−sinθ ) dθ
0

5 π

¿−
3
3

[ ]
2 a ( cos θ )
5
2
2 2

0
4 3
¿− a [ 0−1 ]
15

4 3
A ý= a
15

4 3
a
15
ý= 2
a

4
ý= a
15

By symmetry, x́=0

4
(
Thus, centroid is at 0 ,
15 )
a answer

3
2. Thea area bounded by the curve r =a(1+sin θ). A= π a 2
4

Figure 8. 9
Solution:
π π
2 2
1
A x́= ∫ r cos θ r 2 dθ= 13 a3 ∫ ( 1+sin θ )3 cos θ dθ
2 −π −π
2 2

4 π
( 1+sin θ )
1
¿ a3
3 [ 4 ]2
−π
2

1 3 4 16
¿ a [ ( 2 ) −0 ]= a3
12 12

4
A x́= a3
3
4 3
a
3
Thus, x́=
3
π a2
4

16 a
¿
9 π

16 a
x́=

π
2
1
A ý= ∫ r sin θ r 2 dθ
2 −π
2

π
2
1 3
¿ a3 ∫ ( 1+sin θ ) sin θ dθ
3 −π
2

1 15
¿ a3
3 8 [ ]
π

5
A ý= π a3
8

5 3
πa
8
Thus, ý=
3
π a2
4

5
ý= a
6

Therefore, centroid is at ( 169 πa , 56 a)answer


8.8. Theorem of Pappus

The following theorem, known as the Second Proposition of Pappus , is useful in a


variety of ways.
THEOREM 27. The volume of any solid of revolution is equal to the generating area
times the circumference of the circle described by the centroid of the area.

Figure 8.10 Figure 8.11


For definiteness, take the axis of revolution as x−axis , and suppose first that this forms
part of the boundary of the rotating area (Figure 8.10). The proof is very easy.
b b
2
V =π ∫ y dx , A ý=∫ ydx ,
a a
so that
V =2 π A ý= A ∙2 π ý
But this last formula, translated into words, is the theorem.
The proof is readily extended to cover the situation shown Figure 8.11.

Solved Problems:
1. Find the centroid of a right triangle.

Solution:

Figure 8. 12
V
V =2 π x́ A x́=
2 πA

1 2
πb h
3
Thus, x́=
1
2 π bh
2

1
x́= b
3

Figure 8. 13
V
V =2 π ý A ý =
2 πA

1 2
πh b
3
Thus, ý=
1
2 π bh
2

1
ý= h
3

Therefore, centroid is at ( 13 b , 13 h ) answer


2. Find the centroid of a semicircular area.

Solution:

By symmetry, x́=0 .

For ý :

V =2 π ý A

V
Thus, ý=
2 πA

4 3
πa
3 Figure 8. 14
¿
1
2 π π a2
2

4a
ý=

4a
Therefore, centroid is at 0 , ( 3π )
answer

8.9. Centroid of a Solid of Revolution

Since the centroid of a solid of revolution lies on the axis, a single coordinate determines
its position. If say, the revolution takes place around the x−axis , the general formulas of 8.5
reduce to
V x́=∫ x c dV ,
where of course dV is a disk, ring, or shell, according to convenience, and x , is the x -coordinate
of the centroid of the volume element.

8.10. Centroids of Miscellaneous Solids

Example(a). Find the centroid of one-half of the solid in the example in 7.5.
With the triangular element, we know that the centroid of the element is
2 1
(
( xc , yc , zc )= x , 3 y , 3 z ). Thus
a
1
V x́=∫ x ∙ yz dx
0 2
But z= y , xdx=− ydy :
a
−1
V x́= ∫ x y 2 dx
2 0
a
1
¿− ∫ y 3 dy
2 0
1
¿ a4
8
1 3
In 7.5 we found V = a , so that
3
1 4
a
8 3
x́= = a
1 3 8
a
3
Next,
a
2 1
V ý=∫ y ∙ yz dx
0 3 2
a a 2
1 3 1 2 2 3
¿ ∫ y dx= ∫ ( a −x ) dx
30 3 0
Putting x=a sin θ , we find
1
V ý= π a4
16

1
π a4
16 3
ý= = πa
1 3 16
a
3
1 1 1
Since for all elements z c = z= y = y c,
3 3 2
1 3
ź= ý= πa
2 32
Solved Problems:
1. Hemisphere of radius R. Find the centroid.

Solution:

dV =2 πyxdy

x
V x́=∫ dV
2

x
¿ ∫ 2 πyxdy
2
R
¿ π ∫ x 2 ydy
0

R
¿ π ∫ ( R 2− y 2 ) ydy
0

2 2 2 R

¿−
2 [
π (R −y )
2 ] 0

π 4
¿ R
4

π 4
R
4
Thus, x́=
2
π R3
3

3
x́= R ¿ the center answer
8

2. The volume formed by revolving about Oy the area in the first quadrant bounded by
y 2=4 ax , y=0, x=a . Find the centroid.

Solution:

By symmetry, x́=0 , ź=0

dV =π ( a2−x 2 ) dy

Figure 8. 16

y
V ý=∫ 2 πydx=π ∫ y 2 xdx
2
a
¿ π ∫ 4 ax xdx
0

a
¿ 4 πa ∫ x 2 dx
0
a
x3
¿ 4 πa[ ]
3 0

4 π a4
V ý=
3

4
π a4
3
Thus, ý=
8
π a3
5

5
ý= a
6

5
(
Therefore, centroid is at 0 ,
6 )
a answer

8.11. Centroid of an Arc; of a Surface of Revolution

The centroid of a curved arc or of a surface of revolution can be found by choosing an


element in 7.6 or 7.7 respectively.

9 ITERATED INTEGRALS
9.1 Iterated Integration
The operation of integration may be iterated and thus yields another valuable tool as we
shall see in § 9.3-10.7. Consider a definite integral
x2

(1) ∫ f ( x , y ) dx
x1

in which the integrand is a function of the two variables x and y, either or both of which may be
absent from f(x, y) in specific problems. The limits of integration x 1 and x2 must never depend
upon x, the variable of integration. We may permit x 1 and x2 to depend upon y. Thus
x 1=x 1 ( y)∧x2 =x2 ( y ), although in special cases either of or both x 1 and x2 may be independent
of y. The integral (1) so described is a function of y:
x 2( y)

(2) φ ( y )= ∫ f ( x , y ) dx .
x 1( y)

If x1, x2, and f are reasonably well-behaved functions of y, φ ( y ) will itself be an integrable
function, and we may well consider an integral
b
(3) I =∫ φ ( y ) dy ,
a

in which a and b must, of course, be independent of y.


It is customary to combine (2) and (3) and write
b x2

(4) I =∫ ∫ f ( x , y ) dx dy
a x1

to mean that f(x, y) is to he integrated with respect to x (as indicated by the inner diffeiential dx)
between the limits x1 and x2, and the result of that definite integration to be integrated with
respect to y between the limits a and b.
In evaluating (4) it is important to remember that
(a) During the z (inner) integration the y (other variable of integration) is held constant;
(b) The inner limits of integration x1 and x2 must be independent of x (the inner variable
of integration);
(c) The outer limits of integration a and b must be independent of both variables of
integration x and y.
In §9.3 we shall encounter the first of many instances in which a mathematical or
physical problem leads us naturally to iterated integration. For the moment we concentrate on
obtaining practice in evaluating iterated integrals.
2 y
2 2
Example (a). Evauate ∫ ∫ (x +3 y )dx dy
0 0

We proceed as follows:
2 y 2 x= y
1
2 2
∫∫ ( x +3 y ) dx dy =∫
0 0 0
[ 3
x+ 3 x y 2 ]
x =0
dy

2
¿∫
0
( 13 y +3 y ) dy
3 3

2
10 5 2 5 ∙ 16 40
¿ ∫ y 3 dy= [ y 4 ] 0= = .
3 0 6 6 3
2
2 x
x 2 dx dy
Example (b). Evauate ∫ ∫
0 x ( xy +4 )2
Note that the integration in this example is in the order: y integration first, x integration
second. We obtain
2 2
2 x 2 x
x 2 dx dy −x
∫∫
0 x ( xy +4 )
2 ∫
=
0
xy+ 4[ ] x
dx

2
−x 2 x2
¿∫
0
[ +
x3 + 4 x2 + 4
dx
]
2
4 x2
¿ ∫[ 1− 2 + 3
x +4 x +4
dx
]
0

2
1 1
[
¿ x−2 arctan
2
x− ln ( x 3 + 4 )
3 ] 0

1 1
¿ 2−2 arctan 1− ln 12−0+ 0+ ln 4
3 3
1 1
¿ 2− π− ln 3=0.0630
2 3

Solved Problems:
Evaluate the integral and subsequent iterated integral.
5
1.) (a) ∫ ( 6 x 2 + 4 xy−3 y 2)dy
2

2 5
(b) ∫ ∫ (6 x 2+ 4 xy−3 y 2 )dydx
−3 2

Solution:
5
(a) ∫ ( 6 x 2 + 4 xy−3 y 2)dy=(6 x 2 y +2 x y 2− y 3)¿52
2

¿ 30 x 2+50 x−125−( 12 x 2 +8 x−8 )


¿ 18 x2 + 42 x−117
2 5 2
(b) ∫ ∫ (6 x + 4 xy−3 y )dydx=∫ (18 x 2 +42 x−117) dx
2 2

−3 2 −3

¿( 6 x 3 +21 x 2−117 x) ¿2−3 =48+ 84−234−(−162+189+351)


¿−102−378=480
x
2.) (a) ∫ ( x 2 y− y +2)dy
1

2 x
(b) ∫ ∫ ( x 2 y − y+ 2)dydx
0 1

Solution:
x
x2 y2 y2
(a) ∫ ( x 2 y− y +2)dy=( 2
− +2 y)¿1x
2
1
x 4 x2 x2 1 x4 2 3
¿ − +2 x−( − +2)= −x +2 x−
2 2 2 2 2 2
2 x 2
2 x4 3
(b) ∫ ∫ (x y − y+ 2)dydx=∫ ( − x2 +2 x− )dx
0 1 0 2 2

x5 x3 2 3 32 8
¿ ( 10 3 2 )
− + x − x ¿20 = − +4−3−0
10 3
23
¿
15
y
3.) (a) ∫ ( cos x sin y ) dx
0

π y
(b) ∫ ∫ (cos ⁡x sin ⁡y ) dxdy
0 0

Solution:
y
(a) ∫ ( cos x sin y ) dx= ( sin x sin y ) ¿ 0y =sin2 y
0

π y π π
1−cos ⁡2 y
(b) ∫ ∫ (cos ⁡x sin ⁡y ) dxdy=∫ sin2 ⁡ydy=∫ 2
dy
0 0 0 0

1
y− sin ⁡2 y
2 π π
¿ ¿0π= −0=
2 2 2

9.2 Plane Area by Iterated Integration


An iterated integral with unity as integrand may be interpreted as an area. For instance,
the integral
3 9

∫∫ dx dy
0 y2 may be viewed as follows: We consider an
elementary rectangle of dimensions dx and dy.
First sum those rectangles in the x-direction (y
being held constant) from x = y2, the lower inner
limit of integration, to x = 9, the upper inner limit
of integration. This yields the area of the
horizontal element shown lightly shaded in Fig.
9.1. The horizontal elements are then summed
(integrated) in the y-direction from y = 0 to y = 3.
We thus obtain the area in the first quadrant
bounded by y2 = x, x = 9, y = 0. Indeed,
Figure 9.1

3 9 3
9
A=∫ ∫ dx dy=∫ [ x ] y dy 2

0 y2 0
3 3
1
0 3 [
¿ ∫ ( 9− y 2 ) dy= 9 y − y 3 =18
0
]
Exercises in obtaining areas by iterated integration, in either rectangular or polar
coordinates, are useful for practice in setting up iterated integrals. In presenting such problems,
we are not advocating the use of two integrations when only one is needed.
Example (a). Find the area of a right triangle by iterated integration in rectangular
coordinates. (Fig. 9.2)

The equationFigure 9.2


of the bounding line is Figure 9.3
b
y= x
a
therefore
b
x
a a a b
x
a
A=∫ ∫ dy dx=∫ [ y ] dx 0
0 0 0

a
b 1
¿ ∫ x dx= ab .
a0 2
Example (b). Solve Example (a) in polar coordinates.
From §9.6 we borrow the expression r dr dθ for the element of area in polar
coordinates. The upper limit for r is seen from Fig. 9.3 to be
OP=a sec θ
so that
b b
arctan arctan
a a sec θ a
1
A= ∫ ∫ rdr dθ= a2 ∫ sec 2 θ dθ
0 0 2 0

b
1 2 arctan
a 1 2 b 1
¿ a [ tan θ ] 0 = a ∙ = ab .
2 2 a 2

Solved Problems:
1.) Find the area 𝐴 of the triangle with vertices (1,1) and (3,1), and (5,5) as shown in Figure 9.4.
Figure 9.4
Solution:
The triangle is bounded by the lines as shown in the figure. Choosing to integrate with
y +5
respect to 𝑥x first gives that 𝑥 is bounded by 𝑥 = 𝑦 to  x= , while 𝑦 is bounded
2
by 𝑦=1 to 𝑦=5. (Recall that since x-values increase from left to right, the leftmost curve, 𝑥=𝑦, is
y +5
the lower bound and the rightmost curve,  x= , is the upper bound.) The area is
2
y +5
5 2

A=∫ ∫ dxdy
1 y

5 y+5
2
¿ ∫( x ¿ y )dy
1

5
−1 5
¿ ∫( y + ) dy
1 2 2
−1 2 5
¿( y + y )¿ 51
4 2
¿ 4.
We can also find the area by integrating with respect to 𝑦 first. In this situation, though, we have
two functions that act as the lower bound for the region R, 𝑦=1 and 𝑦=2𝑥−5y. This requires us to
use two iterated integrals. Note how the x-bounds are different for each integral:
3 x 5 x
A ¿ ∫ ∫ 1 dydx +¿ ¿∫ ∫ 1 dydx
1 1 3 2 x−5
3 5 3 5
¿ 2¿=4. ¿
¿ ∫ ( ¿ y ) ¿x1 dx ¿+ ¿ ¿ ∫ (¿ y) ¿2x x−5 dx ¿=∫ (¿ x −1)dx ¿+ ¿ ¿ ∫ (¿−x +5)dx ¿=2 ¿
1 3 1 3

2.) Find the area of the region enclosed by  y=2 x∧ y =x 2, as shown in Figure 9.5.

Figure 9.5
Solution:
Using dy dx :
2 2x 2

∫ ∫ 1 dydx=∫ (2 x−x2 )dx=(x 2− 13 x 3 )¿ 20= 43 .


0 x
2
0

Using dx dy :
4 √y 4

∫ ∫ 1 dxdy=∫ (√ y− y /2) dy=( 23 y 3 /2 − 14 y 2)¿40 = 43 .


0 y /2 0

3.) A graph of a planar region 𝑅 is given. Give the iterated integrals, with both orders of
integration 𝑑𝑦 𝑑𝑥 and 𝑑𝑥 𝑑𝑦, that give the area of 𝑅. Evaluate one of the iterated integrals to find
the area.

Figure 9.6

Solution:
4 7− x 3 y+1 5 7− y
Area=∫ ∫ 1 dydx=∫ ∫ 1 dxdy +∫ ∫ 1 dxdy
2 x−1 1 2 3 2

4 7− x 4 4
7−x
∫∫ 1 dydx=∫ y ¿ x−1 dx=∫ (8−2 x )dx
2 x−1 2 2

¿ ( 8 x−x 2 ) ¿42 =32−16−16+ 4=4 unit s 2

9.3 Volume Under a Surface


The method in §7.5 for finding the volume of a solid succeeds only when the solid can be
cut into slices such that the area of the face of each slice is known. We proceed to develop a
method that is free of this restriction.

Figure 9.7
Consider the solid bounded by a portion R of the surface
z=f ( x , y ) , z ≥0 ,
the area S into which R projects in the xy-plane, and the vertical cylinder through the
boundaries of S and R.
Draw in S a set of n lines parallel to the y-axis and a set of m lines parallel to the x-axis,
thus dividing S into rectangles of area ∆ y ∆ x , together with a number of irregular portions
around the boundary. By passing through each line or the two sets a plane perpendicular to
the xy-plane, we divide V into vertical rectangular columns together with smaller irregular
columns. The upper boundary of each column is a portion of R.
Through that point of the upper boundary of each column which is nearest the xy-plane,
pass a horizontal plane, thus forming a set of rectangular prisms lying wholly within V. As ∆ x
and ∆ y both approach 0, the limit of the sum of all these prisms is the volume under the surface:
n m
(1) V = lim ¿ ∆ x →0 ∑ ∑ f ( x i . y j) ∆ y ∆ x .
∆ y → 0 i=1 j=1
From the critical standpoint this formula, based directly on our intuitive conception of
volume, may be regarded as a definition, analogous to the definition of area. The definition is
valid whenever the limit exists. If z is a continuous function, existence of the limit can be proved.

9.4 Volume Found by Integration


The "double limit" (1) above may be evaluated by two successive applications of
Theorem 19. Let us fix our attention on the rectangle KLMN in S (Fig. 9.7), keeping x and ∆ x
'
constant for the time being. The volume ∆ V i whose base is this rectangle may be found by
adding the volumes of all the included elementary prisms and then taking the limit as ∆ y
approaches zero. Hence, by § 55,
y 'i '

[ ]
m
'
∆ V = lim
i ∑ f ( xi . y j )∆ y ∆ x= ∫ f ( x i . y)dy ∆ x.
m →∞ j =1 '
y i

Here primes are being used to distinguish one y i from another, etc. The primes do not denote
derivatives.

In the expression for ∆ V 'i the coefficient of ∆ x is a function of xi alone, since the limits y' i
and yi'’ are functions of xi alone. Thus we may apply again the theorem of § 55, and find that the
required volume under the surface z = f(x, y) is
y 'i ' b y''

[∫ ] [∫
n
V = lim ∑
n→ ∞ i =1
yi
'
f ( x i . y) dy ∆ x=∫
a y
]
f ( x , y) dy dx ,

where a and b are the extreme values of x on the boundary of S.


The quantity just found is an iterated integral:
b y' '
(1) V =∫ ∫ f (x , y )dy dx .
a y'

Of course we might integrate first with respect to x, then with respect to y. The same
reasoning as before would lead to the formula
d x ''
(2) V =∫ ∫ f ( x , y ) dx dy ,
c x'
y remaining constant during the first integration.
In the foregoing argument we have assumed our solid to be divided into rectangular
columns perpendicular to the xy-plane. Sometimes, however, it is more convenient to erect
columns perpendicular to one of the other coordinate planes. Such variations offer no difficulty,
provided the geometric meaning of the successive integrations be kept clearly in mind. In every
problem a sketch of the required volume should be made and the iterated integral built up by
inspection of the figure.
Any function f(x, y) of two independent variables may be interpreted as the z-coordinate
of a variable point on a surface. If, then, in any problem we can express the required quantity as
a double limit of the form (1), § 9.3, no matter what may be the geometric or physical meaning
of the given function f(x, y), the limit may be evaluated by an iterated integration (1) or (2).
Thus the method described above is by no means confined to the determination of volumes, it
applies to a great variety of problems.
Example (a). Find the volume in the first octant bounded by the plane z=2−x− y and
the cylinder y=1 — x2. (Fig. 9.8)

By means of planes parallel to the yz-plane, cut the solid into thin slabs. Then, by planes
parallel to the zx-plane, cross-cut the slabs into slender vertical columns of base dy dx, height z,
volume z dy dx. (Since we are integrating to integrate first with respect to y, we write z dy dx
rather than z dx dy.) In the first integration, adding up all the columns in the slab, x remains
constant, and y varies from 0 (at P) to 1 — x2 (at Q). The second integration adds up all the
slabs, from x = 0 (at 0) to x = 1 (at A):
2
1 1− x
V =∫ ∫ z dy dx
0 0
2
1 1−x
¿∫ ∫ ( 2−x− y ) dy dx
0 0

1
1 2
1−x
¿− ∫ [ ( 2−x− y )2 ]0 dx
2 0
1
1 2 49
¿− ∫ [ ( 1−x + x 2 ) −( 2−x )2 ] dx=
2 0 60
Example (b). Solve example (a) by second method.
Cut the solid into slabs by planes parallel to the zx-plane, the slabs into columns by
planes parallel to the yz-plane (Fig. 9.9). In the first

Figure 9.8 Figure 9.9


Integration, x varies from 0 (at R) to √ 1− y (at S); then y varies from 0 (at O) to 1 (at B):
1 √1− y 1 √ 1− y
V =∫ ∫ z dx dy=∫ ∫ ( 2−x− y ) dx dy
0 0 0 0

1
1 √ 1− y
¿− ∫ [ ( 2−x− y )2 ]0 dy
2 0
1
1 2 49
¿− ∫ [ ( 2−√ 1− y − y ) −( 2− y )2 ] dy =
2 0 60

Solved Problems:
Evaluate the integral and subsequent iterated integral.
1.) The base of a solid is the triangle in the xy-plane with vertices (0, 0), (1, 0), and (0, 1).
The cross-sections of the solid perpendicular to the x-axis are squares. What is the volume of the
solid?
Solution:

Figure
We integrate with respect to x from x =9.10
0 to x = 1. The equation of the line is y = 1 − x, so the
length of the base of the square is (1 − x) − 0 = 1 − x. Therefore, the area of a cross-section is
given by

A ( x )=( 1−x )2=1−2 x + x 2 .


Hence, the volume of the solid is

2.) Find the volume of the solid obtained by rotating the area between the graphs of y = x 2
and x = 2y around the y-axis.
Solution:

Figure 9.11
2
The two curves intersect when x = x/2, which means either x = 0 or x = 1/2.
Using cylindrical shells, notice that the sides of the cylinder will run from the red line to the blue
x
curve, and so the shells will have height – x 2 . Also, for a given x, the cylinder at x will have
2
radius x − 0 = x, so the volume of the solid is

3.) Find the volume of the solid obtained by rotating the region between the graphs of
y=x √ 2−x and y = 0 around the x-axis.

Solution: Figure 9.12


Since the “washer” is actually just a disk of radius x √ 2− x, we know that the cross-
sectional area is

Now, the blue curve crosses the x-axis when x √ 2− x=0, which happens when x = 0 and x = 2, s
these should be our limits of integration. Hence, the volume of the solid is

9.5 The Double Integal


Being given a function of two independent variables, defined at all points of a plane
region S, let us divide S into k elements ∆ Si (i=1, 2 , … , k) in such a way that as k increases and
∆ Si approaches zero, the maximum distance between any two points on the boundary of ∆ Si
approaches zero. Multiply the area ∆ Si of each element by the value fi of the function at some
point of ∆ Si , add all these products together, and take the limit of the sum. This limit is called

the double integral of f over the region S, and is denoted by the symbol- ∫ ∫ fdS :
S

k
(1) lim ∑ f i ∆ S i=∫∫ fdS .
k → ∞ i=1 S

Let us take a moment to tie this up


with the argument of § 9.3. There, the
independent variables were the rectangular
coordinates (x, y) of any point in S; the
equation
z=f ( x , y )
represented a surface in space; the elements of
area ∆ S were rectangles ∆ y ∆ x; and, since we Figure 9.13
took m cutting planes in one direction and n in
the other, the number of elements was k = mn.
b y' '
The double integral, like the iter-ated integral ∫ ∫ f ( x , y) dy dx may always be
a y'
interpreted as the volume under a surface. Since this .volume, for a given surface, and given base
S, is a definite fixed quantity, the value of a double integral is independent of the mode of
division of S into elements, as long as the longest chord in every ∆ S approaches zero.
Whenever the double integral exists, the iterated integral also exists, and gives us one
means of evaluating the double integral. However, the latter, does not tie us down to a particular
coordinate system or to any particular mode of division of S. We shall take advantage of this in
the next section.

9.6 The Double Integral in Polar Coordinates


Let S be a plane area bounded by a curve
whose equation is given in polar coordinates. We
may divide S into elements ∆ S by means of
concentric circular arcs and radial lines, as in the
figure. Then ∆ S is the difference between two
circular sectors of angle ∆ θ and radii r and r + ∆r
respectively: i.e.,
1 1
∆ S= ( r +∆ r )2 ∆ θ− r 2 ∆θ
2 2
1
¿ r ∆ r ∆ θ+ ( ∆ r )2 ∆ θ
2 Figure 9.14

1
Let f(r,θ ) be a function of the polar coordinates defined at all points of S. Then, since ( ∆ r )2 ∆ θ
2
may be neglected, the.double integral of § 9.5 appears as
∫∫ fdS= ∆lim
r→0
∑ ∑ f ( r ,θ ) r ∆ r ∆ θ .
S
∆ θ→ 0

This double. limit .can be evaluated by two successive applications of Theorem 19 just as, in §
9.4; the result is
β y ''
lim ∑ ∑ f (r , θ)r r ∆ r ∆ θ=∫ ∫ f ( r , θ ) r dr dθ
∆ r →0 a y'
∆θ→0

where a and β are the least and greatest values of e (Fig. 9.14), and r’, r" are the least and
greatest values of r in the typical sector— a and β constant, r’ and r" functions of θ , general.

9.7 Volume in Cylindrical Coordinates


Let (Fig. 9.15)
z=f (r , 0)
be the equation of a surface in cylindrical coordinates (polar coordinates in the xy-plane with
the rectangular z). To find the volume under any portion of this surface, divide the base into
polar elements r dr dθ and on each element a column of height z, volume zr dr dθ . Then
β r ''
lim ∑ ∑ x r ∆ r ∆ θ=∫∫ xr dr dθ .
∆ r →0 a r'
∆θ→0

Figure 9.15 Figure 9.16

Example. Find the volume lying above the triangle bounded by the lines
x=0 , y=x , y=a , and cut off by the surface (Fig. 9.16)
(1) x 2+ y 2+ az=2 a2
In cylindrical coordinates (1) becomes

r 2 +az=2a 2
the upper limit OP for r is found from
y=r sin θ=a ;
π
2 a csc θ
1
V= ∫ ∫ ( 2 a2−r 2 ) r dr dθ
a π 0
4
π
2 a cscθ
1 1
aπ [
¿ ∫ a2 r 2 − r 4
4 ]
0

4

π
2
1 1 2
¿

∫ (
csc 2 θ− csc 4 θ dθ= a3 .
4 3 )
4

Solution:

1.) Find the volume under  z=√ 4−r 2 above the quarter circle bounded by the two axes and
the circle  x 2+ y 2=4 in the first quadrant.

Solution:
In terms of  r and  θ, this region is described by the restrictions 0≤𝑟≤ and 0≤𝜃≤𝜋/, so we
have
¿
2.) Find the volume under  z=√ 4−r 2 above the region enclosed by the
π π
curve r =2cos θ ,− ≤θ ≤ ; See Figure 9.17.
2 2

Figure 9.17
Solution:
The double integral is
π /2 2cos ⁡θ π /2 2 cos ⁡θ

∫ ∫ √ 4−r 2 rdrdθ=2 ∫ ∫ √ 4−r 2 rdrdθ .


−π / 2 0 0 0

We can rewrite the integral as shown because of the symmetry of the volume; this avoids a
complication during the evaluation. Proceeding:
¿
3.) Find the area outside the circle 𝑟=2 and inside 𝑟=4sin𝜃; see Figure 9.18.

Figure 9.18
Solution:
The region is described by 𝜋/6≤𝜃≤5𝜋/6π and 2≤𝑟≤4sin𝜃, so the integral is
¿

9.8 Evaluation by Inversion of Order


We have seen that inversion of the order of integration frequently affords a useful check
on the value of an iterated integral. There are important integrals which cannot be evaluated in
terms of elementary functions, as they stand, but which yield to elementary methods, when the
order of integration is inverted.
1 y
2
y
Example. Evaluate ∫ ∫ e dydx .
0 0

Here the first integration is impossible by elementary methods. We shall invert the
order of integration. The inner integration runs from y = x to y = 1; the outer one from x = 0 to x
= 1. Therefore the integration covers the triangle bounded by x = 0, y = x, y = 1 (Fig. 9.19). The
integrand is unaffected by a change of order of integration. Hence
1 y 1 y
2 2
y
∫∫ e dydx=∫ ∫ e y dxdy
0 0 0 0

1
2 y
¿ ∫ [ e y ]0 dy
0

1
2

¿ ∫ y e y dy
0
Figure 9.19
2 1
1 y
[ ]
¿ e
2 0

1
¿ ( e−1 ) .
2

9.9 Evaluation by Change of Coordinate System


An integration impossible, by elementary methods, in rectangular coordinates, may
become possible (even simple) in polar coordinates, or vice versa. We shall use two examples, of
which the second is of vital importance in many advanced applications of mathematics to
engineering and physics.
Example (a). Evaluate
a √ a2− x2
∫ ∫ √ x 2− y 2 dydx .
0 0

In this example inversion of order of


integration is useless; it would merely interchange
the letters x and y throughout. We can perform the
first integration, but not the second one by
elementary means. The integration runs, from y =
0 to √ a2−x 2, then from z = 0 to x = a; i.e., over the Figure 9.20
first quadrant of a circle of radius a, center at the
origin. This suggests polar coordinates. The new element of area is r dr dθ , and √ x 2− y 2
becomes r. Therefore,
π
a √ a2− x2 2 a

∫ ∫ √ x 2− y 2 dydx=∫∫ r ∙ r dr dθ
0 0 0 0

π
2
1 3 π a3
¿ ∫ a dθ= .
0 3 6

2
−x
Example (b). Evaluate ∫ e dx .
0

This involves only a single integration but a nonelementary one. Put



2

B=∫ e−x dx .
0

Then, of course, it is also true that



2

B=∫ e− y dy .
0

Now consider the iterated integral


∞ ∞
2 2

∫∫ e−x − y dx dy .
0 0
2 2 2 2
Since e− x − y =e−x −e− y , we may write
∞ ∞ ∞ ∞ ∞

∫∫ e
0 0
−x 2− y2
dx dy .=∫
0
[ ∫e
0
− x2
]
dx e
− y2
dy=B∫ e
0
− y2
dy=B .
2

Hence
∞ ∞
2 2

B2=∫ ∫ e− x − y dx dy .
0 0

The region of integration in this last integral is the entire first quadrant. Turning to polar
2
coordinates, we find that the element of area is r dr dθ and the integrand e−r . Therefore,
π
2 ∞
2

B2=∫ ∫ e−r r dr dθ
0 0

π
2 ∞
¿∫
0
[ −1 −r
2
e
2

] 0

π
2
1 1
¿ ∫ dθ= π .
0 2 4
2 1 1
Since B = π and B > 0, B= √ π . That is,
4 2

∫ e−x dx = √2π .
2

Solved Problems:
1.) Reverse the order of integration in the iterated integral then evaluate the new iterated
integral.
2
x= √ 2 y=2−x
2

∫ ∫ x e x dydx .Solution:
x=0 y=0

The region as presented is of Type I. To reverse the order of integration, we must first express
the region as Type II. Refer to figure 9.21.

Figure 9.21
We can see from the limits of integration that the region is bounded above by  y=2−x 2 and
below by 𝑦=0 where 𝑥 is in the interval [0,√2]. By reversing the order, we have the region
bounded on the left by 𝑥=0 and on the right by  x=√ 2− y  where 𝑦 is in the interval [0,2]. We
solved  y=2−x 2in terms of x to obtain  x=√ 2− y .

Hence Reverse the order of integration then use substitution.


2
√2 2−x 2 √2− y
2 2

∫∫ x e x dydx ¿ ∫ ∫ x e x dxdy ¿
0 0 0 0

2.) Consider the itterated integral

∬ f (x , y) dxdy
R

where  z=f ( x , y )=x−2 y over a triangular region 𝑅 that has sides on 𝑥=0, 𝑦=0, and the
line 𝑥+𝑦=1. Sketch the region, and then evaluate the iterated integral by
(a) integrating first with respect to y and then
(b) integrating first with respect to x.
Solution:
A sketch of the region appears in Figure 9.22
Figure 9.22
(a) Integrate 𝑦 from 𝑦=0 to 𝑦=1−𝑥 vertically and then integrating 𝑥 from 𝑥=0 to 𝑥=1:
x=1 y=1− x x=1
y=1−x
∬ f ( x , y ) dxdy= ∫ ∫ ( x−2 y ) dydx= ∫ [ xy −2 y 2 ] y=0 dx
R x=0 y=0 x=0

x=1

∫ ¿¿
x=0

3 2 x=1 −1
¿ [−x + x 2− x 3 ] = .
2 3 x=0 6
(b) Integrate x from x=0 to x=1−y vertically and then integrating 𝑥 from y=0 to y=1:
y=3 x= y+3
2 2
∬ (3 x + y )dA= ∫ ∫ (3 x2 + y 2 ) dxdy
D y=−2 x= y 2−3

y=3
¿ ∫ (x 3 + x y 2 )∨¿ y+3
2
y −3
dy ¿ Iterated integral, Type II region.
y=−2

y=3
¿ ∫ ¿¿
y=−2

3
¿ ∫ (54 +27 y−12 y 2+ 2 y 3 +8 y 4− y 6 )dy Integrate with respect to x
−2

27 y 2 3 y 4 8 y5 y 7 3
¿ [54 y + −4 y + + − ]
2 2 5 7 −2
2375
¿ .
7
(c) Reverse the order of integration and evaluate the integral:
1 √x x

∫ ∫ e y dydx
0 x

Solution:
Sketch the two curves to visualize it.
Figure 9.23

For a given y value we see that x goes from  y 2 to y. So the integral (with reversed order)
becomes:
1 y x
y
∫∫ e dxdy
0 y
2

Now evaluating the integral:


1 y x
y
∫∫ e dxdy
0 y
2

1 x
¿ ∫ (e y y )yy dy
2

2
1 y y
( ) (
y
¿ ∫ e y − e y dy y )
0

1
¿ ∫ ( ey)−(e y y)dy
0

For the second half of the integral we need to use by parts: u= y ,dv =e y hence du=1 , v =e y.
1
e y2 1
¿( ) −( y e y )10+∫ e y dy
2 0 0

e
¿ −e+ e−1
2
e
¿ −1
2

9.10 Area of a Surface


We have seen (§ 7.7) that the area of a surface of revolution may be found by simple
integration. To find areas of curved surfaces in general, double integration is required.
Consider a region R on the surface
(1) z=f ( x , y ).
Let us pass through the boundary of R a vertical
cylinder, cutting from the xy-plane a region S—i.e., S is
the horizontal projection of R. Divide S into elements
∆ Si in any convenient way (§ 9.4), and denote by ∆ Ri
the portion of R lying above the ith element. Draw the
tangent plane at some point (any point) of ∆ Ri ' and
denote by ∆ Ri , the element of area (above ∆ Si ) on the
tangent plane. Now the two elements ∆ Ri and ∆ Ri ' , on
the surface and on the tangent plane, respectively, are
essentially equal in the sense that in taking the limit of
the sum, we may substitute the latter for the former.
Figure 9.24
It is known from geometry (or if not known, is
easily established—Ex. 30 below) that if two planes
intersect at an acute angle, an area in one plane may be
projected into the other by multiplying by the cosine of the included angle. The angle between
the xy-plane and the tangent plane equals the angle between their normals—i.e., it is the angle
between the z-axis and the normal to the surface at the point of contact of the tangent plane.
Thus if γ i is the z-direction angle of the normal Ni,

∆ R'i cos γ i=∆ S i ,


or

∆ R'i =sec γ i ∆ S i .
Adding all the elements and taking the limit of the sum, we define the area R as

(2) R=∫ sec γ dS .


S

Of course if it is more convenient to project the area into the yz or zx-plane, the same
formula holds with α ∨β in place of γ .
−δz δz
The direction cosines of the normal to the surface (1) are proportional to , , 1, so
δx δy
that
1
cos γ = .
δz 2 δz 2
√( δx)( )
+
δy
+1

Thus (2) becomes in rectangular coordinates


b y' '
δz 2 δz 2
(3) R=∫ ∫
a y' √( δx )( )
+
δy
+1 dy dx .

Exampk (a). Find the area of the cylinder* (Fig. 9.25)

2 az=2 a2−ax− y 2
intercepted in the first octant by the planes y = x, y = a.
We find
δz −1 δz − y
= , = .
δx 2 δy a
In the order x,y (which is in this case much simpler),
a y
1 y2
R=∫ ∫
0 0

a y
√ + +1 dx dy
4 a2

1
¿ ∫ ∫ √ 5 a2+ 4 y 2 dx dy
2a 0 0
a
1
¿ ∫ √ 5 a2+ 4 y 2 y dy ,
2a 0
so that
3 a 3
R=
1 2 [
∙ ( 5 a2 + 4 y 2 ) 2 0 =
16 a 3
1
24
]
27−5 2 a2 .( )
*Sections by planes y = k are easily seen to be parallel straight lines, the yz-trace is a
parabola, so that the surface is parabolic cylinder with generators Parallel to the zx-plane.
To use (2) in polar coordinates, we would usually work out sec γ in rectangular coordinates and
transform the tesult to the polar system.

Figure 9.25 Figure 9.26

Example (b). Solve the problem above in polar coordinates.


Using the, value of sec γ from Example (a), we have (Fig. 9.26)

1
R= ∫ ∫ √ 5 a2 + 4 y 2 dS
2a S
π
2 a cscθ
1
¿ ∫
2a π
∫ √5 a 2+ 4 r 2 sin2 θ r dr dθ
0
4

π 3 a cscθ

¿
1 2
∙ ∫
2
[ 2
( 5 a + 4 r sin θ ) 2 2 2
]0

16 a 3 π sin 2 θ
4

3 π 3
( 27−5 ) a 2 2 2
2
csc θ dθ=
( 27−5 ) a 2 2
¿
24
∫ 24
π
4

9.11 Surfaces of Revolution


We defined the area of a surface of revolution; in § 9.10 we defined curved area in
general, including the former as a special case. Thus, for the surface of revolution, area has been
defined in two ways; this is allowable only if the two definitions are equivalent. It is interesting
and not difficult to see how the general formula works out in the special case.
Let a surface of revolution be formed by revolving about Ox, from z = a to x = b, the curve
y = u, t = 0
where u is a function of x. By the area of a surface of revolution, the area generated is
b
du 2
S=2 x ∫ y ds=2 π ∫ u 1+
C a √ ( )
dx
dx .

Any point Q of the given curve describes, in the rotation, a circle of radius MQ=u. Let P:(x, y, z)
be a random point of that circle. Then, directly from the figure,

y 2 + z 2= M´P2= M´Q2=u 2,
or, above the xy-plane,

z=√ u2− y 2 .
This gives
du
u
δz dx
δz −y
= , = .
δx √ u 2− y 2 δy √ u 2− y 2

Substituting in (3), § 9.10, and remembering


that the first octant contains only one-fourth of
the surface, we find after a trifle of simplifying

du 2
b u
R=4 ∫ ∫
a 0
√ 1+ ( )
dx
2
√u − y
dy dx
2
Figure 9.27

b 2 u
du y
¿ 4 ∫ u 1+
a

b

dx ( )[ arcsin ]
u 0
dx

du 2
¿ 2 π ∫ u 1+
a dx √ dx=S . ( )

Solved Problems:

1.) Compute the flux of  F=⟨ x , y , z 4 ⟩ across the cone  z=√ x 2+ y 2 , 0 ≤ z ≤1, in the
downward direction.
Solution:
We write the cone as a vector function: r =⟨ v cos u , v sinu , v ⟩ 0 ≤ u≤ 2 π and 0 ≤ v ≤1.
Then r u =⟨−v sin u , v cos u , 0 ⟩ and r v =⟨ cosu , sinu , 1 ⟩ and r u ×r v =⟨ v cos u , v sin u ,−v ⟩. The
third coordinate –v is negative, which is exactly what we desire, that is, the normal vector points
down through the surface. Then
2π 1 2π 1
4
∫ ∫ ⟨ x, y ,z ⟩ ⋅⟨ v cos ⁡u , v sin ⁡u ,−v ⟩ dvdu ¿ ∫ ∫ xv cos ⁡u+ yv sin ⁡u−z 4 vdvdu
0 0 0 0
2π 1
π
¿ ¿ ∫ ∫ v 2−v5 dvdu= .
0 0
3
2.) Evaluate  ∬ 2 ydS where S is the portion of  y 2 + z 2=4  between x=0 and x=3−z.
S

Solution:
Sketch the surface

Figure 9.28

→ → →
∬ f ( x , y , z )dS=∬ f ( r (u , v ))‖ r u × r v ‖dA
S D

The parameterization is,



r ( x ,θ )=⟨ x , 2sin θ , 2 cos θ ⟩ 0 ≤ θ ≤2 π , 0≤ x ≤ 3−z =3−2 cos ⁡θ
Compute the cross product.
→ →
r x = ⟨ 1,0,0 ⟩ r θ =⟨ 0,2cos ⁡θ ,−2 sin ⁡θ ⟩
→ → →
→ → i j k → →
r x × r θ =¿ 1 0 0 ∨¿ 2sin ⁡θ j +2 cos ⁡θ k
0 2cos ⁡θ −2 sin ⁡θ
The magnitude of the cross product is,
→ →
‖ r x × r θ ‖=√ 4 sin 2 θ+ 4 cos 2 θ=2
The integral is then,

∬ 2 ydS=∬ 2(2 sin ⁡θ)(2) dA=∬ 8 sin ⁡θdA


S D D

2 π 3−2 cos ⁡θ

∬ 2 ydS ¿∫ ∫ 8 sin ⁡θdx dθ


S 0 0

¿ ¿∫ 24 sin ⁡θ−16 sin ⁡θ cos ⁡θdθ


0
¿ ¿

3.) Evaluate ∬ xzdS where S is the portion of the sphere of radius 3 with x ≤ 0 , y ≥ 0∧x ≥ 0
S
.
Solution:

Figure 9.29
→ → →
∬ f ( x , y , z )dS=∬ f ( r (u , v ))‖ r u × r v ‖dA
S D

The parameterization is,



1 1
r (θ , φ)=⟨ 3 sin φ cos θ ,3 sin φ sin θ , 3 cos φ ⟩ π ≤θ ≤ π ,0 ≤ φ ≤ π
2 2
Compute the cross product.
→ →
r θ=⟨−3 sin φ sinθ , 3 sin φ cos θ , 0 ⟩ r φ =⟨ 3 cos ⁡φ cos ⁡θ ,3 cos ⁡φ sin ⁡θ ,−3sin ⁡φ ⟩
→ → →
→ → i j k
r θ × r φ =¿ −3 sin ⁡φ sin ⁡θ 3 sin ⁡φ cos ⁡θ 0 ∨¿
3 cos ⁡φ cos ⁡θ 3 cos ⁡φ sin ⁡θ −3 sin ⁡φ
→ → → →
¿−9 sin 2 φ cos ⁡θ i −9 sin ⁡φ cos ⁡φ sin 2 θ k −9 sin ⁡φ cos ⁡φ cos 2 θ k −9 sin 2 φ sin ⁡θ j
→ → →
¿−9 sin2 φ cos ⁡θ i −9 sin 2 φ sin ⁡θ j −9 sin ⁡φ cos ⁡φ(sin 2 θ+cos 2 θ) k
→ → →
¿−9 sin 2 φ cos ⁡θ i −9 sin2 φ sin ⁡θ j −9 sin ⁡φ cos ⁡φ k
The magnitude of the cross product is,

→ →
‖ r θ × r φ ‖ ¿ √(−9 sin2 φ cos ⁡θ)2+(−9 sin 2 φ sin ⁡θ)2 +(−9 sin ⁡φ cos ⁡φ)2
2 2 2
¿ ¿ √ 81 sin φ(sin φ+ cos φ)
¿ ¿
The integral is then,

∬ xzdS=∬ (3 sin ⁡φ cos ⁡θ)(3 cos ⁡φ)(9 sin ⁡φ)dA=∬ 81cos ⁡φ sin2 φ cos ⁡θdA
S D D

∬ xzdS=∬ 81 cos ⁡φ sin2 φ cos ⁡θdA


S D
1
π
π 2

¿ ∫ ∫ 81cos ⁡φ sin2 φ cos ⁡θdφ dθ


1 0
π
2

π 1
π
¿ ∫ (27 sin 3 φ cos ⁡θdφ)∨¿02 dθ ¿
1
π
2

π
¿ ∫ 27 cos ⁡θdθ
1
π
2

¿( 27 sin ⁡θ)∨¿ π1 =¿ ¿
π
2

10 TRIPLE INTEGRALS
10.1 The Triple Integral in Rectangular Coordinates
Suppose we have given a continuous function f(x,y,z) defined at all points of a three-
dimensional region V. Let us pass through V three sets of planes parallel to the coordinate
planes, thus dividing V into elementary
boxes of volume ∆ x ∆ y ∆ z , together with smaller irregular portions around the boundary. Now
multiply the volume of each element by the value of the function at some point of the element,
and form the sum of these products. The triple limit

lim ∑ ∑ ∑ f (x , y , z) ∆ x ∆ y ∆ z
∆ z →0
∆ y→ 0
∆ x →0

is defined as the value of the triple integral, or volume integral, of f(x,y,z) throughout the region
V.
This limit may be evaluated by three successive integrations:

T = lim ∑ ∑ ∑ f (x , y , z) ∆ x ∆ y ∆ z
∆ z →0
∆ y→ 0
∆ x →0

b y ' ' x' '


(1) ¿ ∫ ∫ ∫ f ( x , y , z )∆ x ∆ y ∆ z .
a y ' z'

The first integration extends over a vertical column of base ∆ y ∆ x ; the limits z’,z’’ are the
extreme values of z in this column and in general are functions of both x and y. The integration
with respect to y is extended over a slice parallel to the yz-plane; the limits y; and y’’ are the
extreme values of y in this slice and are functions of x alone. In the final integration the limits
are, of course, the extreme values of x in the whole region.
Since there are six permutations of the three letters x, y, z, five other orders of
integration are possible, in addition to (1).

10.2 Volume as a Triple Integral


If in 10.1 the given function be taken as unity, the integrand becomes merely the volume
element, so that the result of integration is the volume itself:
b y' ' x ' '
V =∫ ∫ ∫ dz dy dx .
a y ' z'

Example. Fine the volume sliced off from the


Paraboloid x 2+ z2 =ay by the plane y = a. (Fig 10.2)
We read directly from the figure
a a √ ay−x 2
V =4 ∫ ∫ ∫ dz dy dx
0 x2 0
a

a a
2

¿ 4 ∫ ∫ [ z ] √0 ay−x dy dx
0 x2
a

Figure 10.2
a a
¿ 4 ∫ ∫ √ ay−x 2 dy dx
0 x2
a

a 2 a

¿
8
3a 0
[
∫ ( ay −x2 ) 3 ] 2
x
a
π
dx= a3 .
2
For practice in reading limits,
The student should verify the following integrals for the same volume:
a √ ay √ ay− x 2
V =4 ∫ ∫ ∫ dz dy dx ;
0 0 0

a √ a2−x 2 a
V =4 ∫ ∫ ∫ dy dz dx .
0 0 x +z
2 2

A figure should be drawn for each case.

Properties of Triple Integrals


If F = F(x, y, z) and G = G(x, y, z) are continuous, then

10.3 The Triple Integral: General Formulation


We may generalize the setup of 10.1 in two ways. First, let the given function f be a
function, not necessarily of the rectangular x, y, z, but of any three independent variables.
Second, let the region of integration be divided into k volume elements ∆ V i of any shape
whatever, subject to the single condition that as k increases, the maximum distance between
any two points of ∆ V i approaches zero.

Now multiply each ∆ V i by the value of f at some point of the element, and add all these
products. The limit of this sum as k approaches infinity (always provided limit exists) is the
triple integral of f throughout V;
k
(1) lim ∑ f i ∆V i=∬ ∫ fdV .
k → ∞ i=1 V

10.4 The Triple Integral in Cylindrical Coordinates


Divide the volume into elements by planes through the z-axis, cylinders around the

z-axis, and planes perpendicular to the z-axis (Fig


10.3). Then the base of the element is r ∆ r ∆ θ , the
altitude ∆ z , volume r ∆ z ∆ r ∆ θ, so that
lim ∑ ∑ ∑ f ( r ,θ , z ) ∆ z ∆ r ∆ θ
∆ z→0
∆ r →0
∆θ→0
β r' ' z' '
¿ ∫ ∫ ∫ f (r ,θ , z)∆ z ∆ r ∆ θ .
a r' z'
In particular, if
f ( r , θ , z )=1 ,
the integral represents the volume of the region in
question:
β r '' z ''
V =∫ ∫ ∫ r dz dr dθ .
Figure 10.3 a r' z'

Example. Find the volume in the first octant inside the cylinder x 2+ y 2=ay and the
paraboloid x 2+ y 2+ az=a2 . (Fig 10.3)
In cylindrical coordinates, the given equations are

r =a sin θ , r 2+ az=a2 ;
2 2
π a −r
2 a sinθ a
V =∫ ∫ ∫ r dz dr dθ
0 0 0

π π
2 2
2 a sinθ a −r 2 a sinθ
1
¿∫ ∫ [z] 0
a
r dr dθ= ∫ ∫ (a2−r 2)r dr dθ
0 0 a 0 0

π π
2 3 2
a sin θ
1 2 2 a 5
¿− ∫
4a 0
[ ( a 2−r ) ]0 dθ=
4
∫ ( 1−cos 4 θ ) dθ= 64 π a2 .
0

Coordinate Conversion Formulas


Solved Problems:
1.) Set up the limits of integration for evaluating the triple integral of a function F(x, y, z) over
the tetrahedron D with the vertices (0,0,0), (1,1,0), (0,1,0), and (0,1,1).
Solution: We sketch D along with its “shadow” R in the xz-plane
The integral is

Alternative way: Using the Order dz dy dx


The integral is

Figure 10.4
2.) Find the volume of the region D enclosed by the surfaces z=x 2 +3 y 2∧z=8−x 2− y 2 .
Solution: The volume is

Figure 10.5
3.) Find a spherical coordinate equation for the sphere x 2+ y 2+ ( z −1 )2=1.

Solution:

x 2+ y 2+ ( z −1 )2=1

ρ2=2 ρ cos ϕ
ρ=2cos ϕ .
Figure 10.6

4.) Find the volume of the “ice cream cone” D cut from the solid sphere ρ ≤1by the cone ϕ=π /3.

2
Solution: The volume is V =∭ ρ sin ϕ dρ dϕ dθ , the integral f ( ρ , ϕ , θ )=1 over D.
D

10.5 Heterogeneous Masses Figure 10.7

The density of a homogenous mass has been defined as the ratio of the mass to the
volume it occupies:
M
δ=
V
For a heterogeneous mass, i.e., one whose density varies from point to point, we must introduce
the idea of density at a point.
Consider an element of volume ∆ V including a point P, and let ∆ M denote the mass
∆M
contained in ∆ V . Then the ratio is the average density of ∆ V . If ∆ V approaches zero in
∆V
∆M
such a way that P is always included, the ratio in general approaches a limit δ , called the
∆V
density at the point P:
∆ M dM
δ = lim = .
∆ V →0 ∆ V dV
If the density at any point is given as a function of the coordinates, the mass can be
found by integration. In the most general case we divide the space ocupied by the body into
volume elements ∆ V i as in 10.3, multiply each element by the density δ i at one of its points, and
all these products. The limit of this sum is the mass:

lim ∑ ∑ ∑ ∆ δi ∆ V i=0.
∆ V i→ 0

That is, the variation inδ throughout the element makes a zero contribution to the mass
integral. It is important to see two points clearly:
(a) The element must always be homogenous, since otherwise, building up the integral,
we would not know what value to use for δ .
(b) Homogeinity of the element is the only requisite: The volume element may be of any
character whatever, provided the mass contained in it is homogenous, since then we know the
values of both δ ∧dV .
If δ varies in some simpler manner, it is possible in many cases to find a homogenous
element of one of the shapes used in Chapter 7 and 9; if so, we may find the volume by simple or
at worst by double integration.
Example (a). Find the mass of a circular cone whose density varies as the distance from
the axis (Fig 10.8)
Let the cone be generated by revolving the line
x y
+ =1
h a
about Ox. If we divide the mass into cylindrical shells about the axis, each element will be
homogenous of density δ =ky :
a
M =∫ δdV =2 πk ∫ y ∙ yx dy
0

a
hy 1
¿ 2 πk ∫ y 2 (h− )dy = πk a3 h.
0 a 6
Figure 10.8 Figure 10.9

Example (b). Find the mass of a circular plate if the density is proportional to the sum of
the distances from two perpendicular diameters. (Fig. 10.9)
With the two diameters as axes, δ =k (x+ y), whence
a √ a2−x 2
M =4 k ∫ ∫ ( x+ y )dy dx , etc .
0 0

Example (c). Find the mass of a sphere whose density is proportional to the sum of the
distances from three mutually perpendicular diametral planes.
Since the density varies with all three coordinates, a triple integral is required: in
rectangular coordinates,
a √ a2− x2 √ a2− x2− y2
M =8 k ∫ ∫ ∫ ( x + y + z ) dz dy dx .
0 0 0

10.6 Centroids; Moments of Inertia


In Chapter 8, in order to make the ideas of centroid and moment of inertia intelligible on
an intuitive basis, we adopted the rough-and-ready expediment of considering the mass as an
“aggregate of particles”. We are now, for the first time, in position to state analytic definitions of
first and second moments of mass.
Given a three-dimensional mass M occupying a volume V, with the density δ expressed
as a function of the coordinates (including as a special case the homogenous body--- δ constant),
take
∆ M i=δ i ∆ V i
as in §10.5. Multiply each element by the distance x i of one of its points from the yz-plane, and
add all the products. The limit of this sum is defined as the first moment of the mass with
respect to the yz-plane. Similar definitions hold for moments with respect to the zx- and xy-
planes. The point ( x́ , ý , ź) whose coordinates are given by the equations

M x́ =∬∫ xδ dV ,
V

M ý =∬∫ yδ dV ,
V

M ź =∬∫ zδ dV
V
is the centroid of the mass.
In analogous fashion we arrive at a suitable definition for moment of inertia with
respect to the x-axis:

I x =∬∫ ( y 2 + z 2) δ dV ,
V

with similar definitions for Iy and Iz.


For area masses or line masses, the defining formulas are the two- and one-dimensional
analogues of those above.
Example (a). In example (a), §10.5,
a a
x πk h2
M x́ =2 πk ∫ ∙ y ∙ yx dy= 2 ∫ y2 ( a− y )2 dy
0 2 a 0
1
¿ πk a3 h2 ,
30
1
πk a 3 h 2
30 1
x́= = h
1 5
πk a3 h
6
Example (b). In example (b), §10.5,
a √ a2−x 2
I x =4 k ∫ ∫ y 2 (x + y) dy dx .
0 0

10.7 Translation Theorem on Moments of Inertia


We are now able to prove the Translation theorem, that if the lines l and c are parallel at
a distance h apart, and if c passes through the centroid of the mass M, then

I t=I c + M h 3 .
Take the line c as x-axis, the line l as y = h,
z = 0. With a volume element chosen in any suitable
manner (the figure shows only one element), we have

I c =∬ ∫ ( y 2 + z 2 ) δ dV ,
V

I t=∬∫ [ ( h− y )2+ z 2 ]δ dV
V
Figure 10.10
¿ h2∬ ∫ δ dV −2 h∬ ∫ yδ dV +∬ ∫ ( y 2 + z 2 ) δ dV .
V V V

By §10.5, the first of these three integrals is Mh 3; by §10.6, the second is -2hM ý , and therefore
vanishes, since the centroid is in the zx-plane; the third is I c.
COROLLARY. If two parallel lines l1, l2 are at distances h1, h2 from the centroid,
I t 1=I t 1 + M ( h22−h 21) .

Solved Problems:
1.) Suppose that 𝑄 is a solid region bounded by  x +2 y +3 z=6 x +2 y +3 z=6and the
coordinate planes and has density  ρ( x , y , z)=x 2 yz . Find the total mass.

Solution:
The region 𝑄 is a tetrahedron (Figure  10.11) meeting the axes at the points (6,0,0),
(0,3,0), and (0,0,2). To find the limits of integration, let 𝑧= 0 in the slanted plane 
1
z= (6−x −2 y ). Then for 𝑥 and y find the projection of 𝑄 onto the xy-plane, which is bounded
3
by the axes and the line  x +2 y =6. Hence the mass is
x=6 y=1/ 2(6− x) z=1/ 3(6− x−2 y)
108
m=∭ ρ( x , y , z )dV = ∫ ∫ ∫ x 2 yzdzdydx=
Q x=0 y=0 z=0 35

Figure 10.11

2.) Suppose 𝑄 is a solid region bounded by the plane  x +2 y +3 z=6 and the coordinate
planes with density  ρ( x , y , z)=x 2 yz (see Figure 10.11). Find the center of mass using decimal
approximation.
Solution:
1 1
y= (6− x) z= (6− x−2 y)
x=6 2 3
54
M xy =∭ zρ( x , y , z )dV = ∫ ∫ ∫ x 2 y z 2 dzdydx= ≈ 1.543 ,
Q x=0 y=0 z=0 35
1 1
y= (6 −x) z = (6 −x−2 y)
x=6 2 3
81
M xz =∭ yρ(x , y , z)dV = ∫ ∫ ∫ x2 y 2 zdzdydx= ≈ 2.314 ,
Q x=0 y=0 z=0 35
1 1
y= (6− x) z= (6− x−2 y)
x=6 2 3
243
M yz =∭ xρ(x , y , z )dV = ∫ ∫ ∫ x 3 yzdzdydx= ≈ 6.943 .
Q x=0 y=0 z=0 35
Hence the center of mass is
M yz M xz M xy
x= , y= , z= ,
m m m
M yz 243/35 243
x= = = =2.25 ,
m 108/35 108
M xz 81/35 81
y= = = =0.75 ,
m 108 /35 108
M xy 54/35 54
z= = = =0.5
m 108/35 108
The center of mass for the tetrahedron 𝑄 is the point (2.25,0.75,0.5).

3.) Suppose that 𝑄 is a solid region and is bounded by  x +2 y +3 z=6 and the coordinate
planes with density  ρ( x , y , z)=x 2 yz (see Figure 10.11). Find the moments of inertia of the
tetrahedron 𝑄 about the yz-plane, the xz-plane, and the xy-plane.
Solution:
The moments of inertia of the tetrahedron 𝑄 about the yz-plane, the xz-plane, and
the xy-plane are

I x =∭ ( y 2 + z 2) ρ(x , y , z) dV ,
Q

I y =∭ (x 2+ z2 ) ρ(x , y , z )dV ,
Q

and

I z =∭ ( x2 + y 2 )ρ ( x , y , z ) dVwithρ( x , y , z)=x 2 yz .
Q

Proceeding

I x =∭ ( y 2 + z 2) x 2 ρ( x , y , z )dV
Q
1 1
y= (6− x) z= (6− x−2 y)
x=6 2 3
117
¿∫ ∫ ∫ ( y 2+ z2 )x 2 yzdzdydx= ≈ 3.343 ,
x=0 y=0 z=0
35

I y =∭ (x 2 + z 2) x2 ρ(x , y , z)dV
Q
1 1
y= (6− x) z= (6− x−2 y)
x=6 2 3
684
¿∫ ∫ ∫ (x 2+ z 2 )x 2 yzdzdydx= ≈19.543 ,
x=0 y=0 z=0
35
I z=∭ ( x 2 + y 2) x 2 ρ(x , y , z)dV
Q
1 1
y= (6− x) z= (6− x−2 y)
x=6 2 3
729
¿∫ ∫ ∫ (x 2+ y 2 )x 2 yzdzdydx= ≈ 20.829 .
x=0 y=0 z=0
35

Thus, the moments of inertia of the tetrahedron 𝑄 about the yz-plane, the xz-plane, and


the xy-plane are 117/35,684/35, and 729/35, respectively.

REFERENCES
https://www.iitrpr.ac.in/MA101/MA101_LectureNotes(2019-20)Module_12.pdf
https://math.libretexts.org/Bookshelves/Calculus/Book
%3A_Calculus_(OpenStax)/15%3A_Multiple_Integration/15.6%3A_Calculating_Centers_of_Mass
_and_Moments_of_Inertia
https://math.libretexts.org/Courses/University_of_California_Irvine/MATH_2E_Multivariable_C
alculus/Chapter_14%3A_Multiple_Integration/14.2b
%3A_Double_Integrals_Over_General_Regions
https://www.math.colostate.edu/~clayton/teaching/m2260s12/exams/exam1practicesolutio
ns.pdf
https://math.stackexchange.com/questions/1561249/reverse-the-order-of-integration-and-
evaluate-the-integral
https://www.whitman.edu/mathematics/calculus_online/section16.07.html
https://tutorial.math.lamar.edu/problems/calciii/SurfaceIntegrals.aspx
Asin, Ricardo C. “Integral Calculus-Third Edition”. Merriam & Webster Inc.
Love, C.E. and Rainville, E.D. “Differential and Integral Calculus-Sixth Edition”. Macmillan
Publishing Co, Inc.

You might also like